You are on page 1of 181

BASIC MATHEMATICS

David S I Iiyambo
University of Namibia
November, 2008

0.1

Acknowledgements

Usage of questions from Mathematics 1A and Foundation Mathematics past question papers
for exercises is acknowledged. Some questions were also taken from a variety of basic algebra
textbooks. The proofreader is thanked for the time he/she has put in.

Contents
0.1
1

Acknowledgements . . . . . . . . . . . . . . . . . . . . . . . . . . . . . . . . . . .

Sets
1.1 Definition of a Set . . . . . . .
1.2 Cardinality of Sets . . . . . . .
1.3 Subset and Proper subset . . .
1.4 Operations on Sets . . . . . .
1.5 Venn Diagrams . . . . . . . . .
1.6 Intervals . . . . . . . . . . . . .
1.6.1 Real Numbers . . . . .
1.6.2 Intervals . . . . . . . . .
1.6.3 Operations on Intervals
1.7 Summary . . . . . . . . . . . .
1.8 Answers to Exercises in Unit 1

.
.
.
.
.
.
.
.
.
.
.

1
3
4
5
9
11
12
12
14
16
18
19

.
.
.
.
.
.
.
.
.
.

21
24
24
27
34
35
38
40
42
45
46

3 Equations
3.1 Linear Equations . . . . . . . . . . . . . . . . . . . . . . . . . . . . . . . . . . . .
3.2 Simultaneous Linear Equations . . . . . . . . . . . . . . . . . . . . . . . . . . . .

48
51
53

.
.
.
.
.
.
.
.
.
.
.

.
.
.
.
.
.
.
.
.
.
.

.
.
.
.
.
.
.
.
.
.
.

.
.
.
.
.
.
.
.
.
.
.

.
.
.
.
.
.
.
.
.
.
.

.
.
.
.
.
.
.
.
.
.
.

.
.
.
.
.
.
.
.
.
.
.

.
.
.
.
.
.
.
.
.
.
.

.
.
.
.
.
.
.
.
.
.
.

.
.
.
.
.
.
.
.
.
.
.

.
.
.
.
.
.
.
.
.
.
.

.
.
.
.
.
.
.
.
.
.
.

.
.
.
.
.
.
.
.
.
.
.

2 Algebraic Expressions
2.1 Simplification and Expansion of Algebraic Expressions
2.1.1 Addition and Subtraction . . . . . . . . . . . .
2.1.2 Multiplication and Division . . . . . . . . . . .
2.1.3 Factoring . . . . . . . . . . . . . . . . . . . . .
2.1.4 Factoring Quadratic expressions . . . . . . . .
2.2 Rational Exponents and Radicals . . . . . . . . . . . .
2.2.1 Radicals . . . . . . . . . . . . . . . . . . . . . .
2.2.2 Rationalization of Denominator and Numerator
2.3 Summary . . . . . . . . . . . . . . . . . . . . . . . . .
2.4 Answers to Exercises in Unit 2 . . . . . . . . . . . . .

ii

.
.
.
.
.
.
.
.
.
.
.

.
.
.
.
.
.
.
.
.
.

.
.
.
.
.
.
.
.
.
.
.

.
.
.
.
.
.
.
.
.
.

.
.
.
.
.
.
.
.
.
.
.

.
.
.
.
.
.
.
.
.
.

.
.
.
.
.
.
.
.
.
.
.

.
.
.
.
.
.
.
.
.
.

.
.
.
.
.
.
.
.
.
.
.

.
.
.
.
.
.
.
.
.
.

.
.
.
.
.
.
.
.
.
.
.

.
.
.
.
.
.
.
.
.
.

.
.
.
.
.
.
.
.
.
.
.

.
.
.
.
.
.
.
.
.
.

.
.
.
.
.
.
.
.
.
.
.

.
.
.
.
.
.
.
.
.
.

.
.
.
.
.
.
.
.
.
.
.

.
.
.
.
.
.
.
.
.
.

.
.
.
.
.
.
.
.
.
.
.

.
.
.
.
.
.
.
.
.
.

.
.
.
.
.
.
.
.
.
.
.

.
.
.
.
.
.
.
.
.
.

.
.
.
.
.
.
.
.
.
.
.

.
.
.
.
.
.
.
.
.
.

.
.
.
.
.
.
.
.
.
.
.

.
.
.
.
.
.
.
.
.
.

.
.
.
.
.
.
.
.
.
.
.

.
.
.
.
.
.
.
.
.
.

3.3

3.4
3.5
3.6
4

Quadratic Equations . . . . . . . . . . . . . . . .
3.3.1 Solving By Factoring . . . . . . . . . . . .
3.3.2 Solving by Completing the Squares . . . .
3.3.3 Solving by using The Quadratic Formula
Simultaneous Nonlinear Equations . . . . . . . .
Summary . . . . . . . . . . . . . . . . . . . . . .
Answers to Exercises in Unit 3 . . . . . . . . . .

Inequalities
4.1 Linear Inequalities . . . . . . .
4.2 Nonlinear Inequalities . . . . .
4.3 Summary . . . . . . . . . . . .
4.4 Answers to Exercises in Unit 4

.
.
.
.

.
.
.
.

.
.
.
.

.
.
.
.

Absolute Value
5.1 Properties of Absolute Value . . . . .
5.2 Equations Involving Absolute Values .
5.3 Inequalities Involving Absolute Values
5.4 Summary . . . . . . . . . . . . . . . .
5.5 Answers to Exercises in Unit 5 . . . .

.
.
.
.

.
.
.
.
.

.
.
.
.

.
.
.
.
.

Polynomials
6.1 Addition and Subtraction of Polynomials
6.2 Multiplication of polynomials . . . . . . .
6.3 Polynomial Division . . . . . . . . . . . .
6.4 Real Zeroes/Roots of Polynomials . . . .
6.4.1 Factorizing Polynomials . . . . . .
6.5 Summary . . . . . . . . . . . . . . . . . .
6.6 Answers to Exercises in Unit 6 . . . . . .

.
.
.
.

.
.
.
.
.

.
.
.
.
.
.
.

.
.
.
.

.
.
.
.
.

.
.
.
.
.
.
.

.
.
.
.

.
.
.
.
.

.
.
.
.
.
.
.

.
.
.
.

.
.
.
.
.

.
.
.
.
.
.
.

.
.
.
.
.
.
.

.
.
.
.

.
.
.
.
.

.
.
.
.
.
.
.

.
.
.
.
.
.
.

.
.
.
.

.
.
.
.
.

.
.
.
.
.
.
.

.
.
.
.
.
.
.

.
.
.
.

.
.
.
.
.

.
.
.
.
.
.
.

.
.
.
.
.
.
.

.
.
.
.

.
.
.
.
.

.
.
.
.
.
.
.

.
.
.
.
.
.
.

.
.
.
.

.
.
.
.
.

.
.
.
.
.
.
.

Partial Fractions
7.1 Case 1: Denominator with Distinct Linear Factors. . . . .
7.2 Case 2: Denominator with Repeated Linear Factors. . . .
7.3 Case 3: Denominator with Irreducible Quadratic Factors.
7.4 Case 4: Improper Rational Expressions. . . . . . . . . . .
7.5 Summary . . . . . . . . . . . . . . . . . . . . . . . . . . .
7.6 Answers to Exercises in Unit 7 . . . . . . . . . . . . . . .

iii

.
.
.
.
.
.
.

.
.
.
.

.
.
.
.
.

.
.
.
.
.
.
.

.
.
.
.
.
.

.
.
.
.
.
.
.

.
.
.
.

.
.
.
.
.

.
.
.
.
.
.
.

.
.
.
.
.
.

.
.
.
.
.
.
.

.
.
.
.

.
.
.
.
.

.
.
.
.
.
.
.

.
.
.
.
.
.

.
.
.
.
.
.
.

.
.
.
.

.
.
.
.
.

.
.
.
.
.
.
.

.
.
.
.
.
.

.
.
.
.
.
.
.

.
.
.
.

.
.
.
.
.

.
.
.
.
.
.
.

.
.
.
.
.
.

.
.
.
.
.
.
.

.
.
.
.

.
.
.
.
.

.
.
.
.
.
.
.

.
.
.
.
.
.

.
.
.
.
.
.
.

.
.
.
.

.
.
.
.
.

.
.
.
.
.
.
.

.
.
.
.
.
.

.
.
.
.
.
.
.

.
.
.
.

.
.
.
.
.

.
.
.
.
.
.
.

.
.
.
.
.
.

.
.
.
.
.
.
.

.
.
.
.

.
.
.
.
.

.
.
.
.
.
.
.

.
.
.
.
.
.

.
.
.
.
.
.
.

.
.
.
.

.
.
.
.
.

.
.
.
.
.
.
.

.
.
.
.
.
.

.
.
.
.
.
.
.

.
.
.
.

.
.
.
.
.

.
.
.
.
.
.
.

.
.
.
.
.
.

.
.
.
.
.
.
.

.
.
.
.

.
.
.
.
.

.
.
.
.
.
.
.

.
.
.
.
.
.

.
.
.
.
.
.
.

58
60
61
63
65
68
69

.
.
.
.

70
73
74
77
78

.
.
.
.
.

79
81
83
85
87
88

.
.
.
.
.
.
.

89
92
92
93
98
99
102
103

.
.
.
.
.
.

104
107
109
110
112
114
115

Functions
8.1 Definition of a Function . . . .
8.2 Arithmetic with functions . . .
8.3 Even and odd functions . . . .
8.4 Summary . . . . . . . . . . . .
8.5 Answers to Exercises in Unit 8

.
.
.
.
.

.
.
.
.
.

.
.
.
.
.

Trigonometry
9.1 Angle Measure . . . . . . . . . . . .
9.2 Trigonometric Functions . . . . . . .
9.3 Evaluating Trigonometric Functions
9.4 Graphs of Trigonometric Functions .
9.5 Basic Trigonometric Identities . . . .
9.6 Trigonometric equations . . . . . . .
9.7 Summary . . . . . . . . . . . . . . .
9.8 Answers to Exercises in Unit 9 . . .

10 Sequences
10.1 The General Term . . . . . . . . .
10.2 Recursive Definition . . . . . . . .
10.3 Arithmetic Sequence . . . . . . . .
10.4 Geometric Sequence . . . . . . . .
10.4.1 Infinite Geometric Sequence
10.5 Summary . . . . . . . . . . . . . .
10.6 Answers to Exercises in Unit 10 . .
11 The Binomial Formula
11.1 Factorial . . . . . . . . . . . . .
11.2 The Binomial Coefficient . . . .
11.3 The Binomial Formula . . . . .
11.4 Summary . . . . . . . . . . . .
11.5 Answers to Exercises in Unit 11

.
.
.
.
.

.
.
.
.
.

.
.
.
.
.
.
.

.
.
.
.
.

.
.
.
.
.

.
.
.
.
.
.
.
.

.
.
.
.
.
.
.

.
.
.
.
.

iv

.
.
.
.
.

.
.
.
.
.
.
.
.

.
.
.
.
.
.
.

.
.
.
.
.

.
.
.
.
.

.
.
.
.
.
.
.
.

.
.
.
.
.
.
.

.
.
.
.
.

.
.
.
.
.

.
.
.
.
.
.
.
.

.
.
.
.
.
.
.

.
.
.
.
.

.
.
.
.
.

.
.
.
.
.
.
.
.

.
.
.
.
.
.
.

.
.
.
.
.

.
.
.
.
.

.
.
.
.
.
.
.
.

.
.
.
.
.
.
.

.
.
.
.
.

.
.
.
.
.

.
.
.
.
.
.
.
.

.
.
.
.
.
.
.

.
.
.
.
.

.
.
.
.
.

.
.
.
.
.
.
.
.

.
.
.
.
.
.
.

.
.
.
.
.

.
.
.
.
.

.
.
.
.
.
.
.
.

.
.
.
.
.
.
.

.
.
.
.
.

.
.
.
.
.

.
.
.
.
.
.
.
.

.
.
.
.
.
.
.

.
.
.
.
.

.
.
.
.
.

.
.
.
.
.
.
.
.

.
.
.
.
.
.
.

.
.
.
.
.

.
.
.
.
.

.
.
.
.
.
.
.
.

.
.
.
.
.
.
.

.
.
.
.
.

.
.
.
.
.

.
.
.
.
.
.
.
.

.
.
.
.
.
.
.

.
.
.
.
.

.
.
.
.
.

.
.
.
.
.
.
.
.

.
.
.
.
.
.
.

.
.
.
.
.

.
.
.
.
.

.
.
.
.
.
.
.
.

.
.
.
.
.
.
.

.
.
.
.
.

.
.
.
.
.

.
.
.
.
.
.
.
.

.
.
.
.
.
.
.

.
.
.
.
.

.
.
.
.
.

.
.
.
.
.
.
.
.

.
.
.
.
.
.
.

.
.
.
.
.

.
.
.
.
.

.
.
.
.
.
.
.
.

.
.
.
.
.
.
.

.
.
.
.
.

.
.
.
.
.

.
.
.
.
.
.
.
.

.
.
.
.
.
.
.

.
.
.
.
.

.
.
.
.
.

.
.
.
.
.
.
.
.

.
.
.
.
.
.
.

.
.
.
.
.

.
.
.
.
.

.
.
.
.
.
.
.
.

.
.
.
.
.
.
.

.
.
.
.
.

.
.
.
.
.

.
.
.
.
.
.
.
.

.
.
.
.
.
.
.

.
.
.
.
.

.
.
.
.
.

.
.
.
.
.
.
.
.

.
.
.
.
.
.
.

.
.
.
.
.

.
.
.
.
.

.
.
.
.
.
.
.
.

.
.
.
.
.
.
.

.
.
.
.
.

.
.
.
.
.

116
118
127
128
131
132

.
.
.
.
.
.
.
.

133
135
138
141
146
148
151
154
155

.
.
.
.
.
.
.

157
160
160
161
164
167
168
169

.
.
.
.
.

170
171
171
173
174
175

Unit 1

Sets
Content
1.1 Definition of a Set
1.2 Cardinality of Sets
1.3 Subset and Proper subset
1.4 Operations on Sets
1.5 Venn Diagrams
1.6 Intervals
1.6.1 Real Numbers
1.6.2 Intervals
1.6.3 Operations on Intervals

Introduction
Collections of objects of all types are found practically everywhere in life. The moment you have
objects that have common trait (characteristic), you can form what is known in mathematics as
a set of these objects. In this Unit, we will look at the formal definition of a set and how sets
are presented, and also at the terminology used in relation to sets. Since the number of objects
belonging to a set is in most cases informative, we will also look at this concept briefly. There
are some sections in this Unit that deal with operations on sets and how we deal with subsets
of a given set. Other sections put a scope on the Real number system and on the subsets of the
set of Real numbers, referred to as intervals.

Objectives
By the end of this Unit, you should be able to:
define a set, and determine whether two given sets are equal or not.
represent a set properly, concisely and understandably.
distinguish finite sets from infinite sets.
find the cardinality of a given set (if it is finite).
identify subsets of a given set.
perform rigorous operations on sets.
represent sets with, and interpret Venn diagrams.
prove identities involving sets.
show an understanding of the basic structure of the real number system.
represent intervals as sets, and on the number line.

Activities and Study Times


All the activities and studying this Unit should not take you more than six (6) hours.

1.1

Definition of a Set

Definition 1.1.1 In mathematics, a set is a collection of well-defined objects. This means that
one must be able to tell whether a given object belongs to that set or not. The objects that belong
to a set are called the elements or members of the set, and the notation x X denotes the fact
that x is an element of set X, while the notation x
/ X denotes the fact that x is not an element
of set X.

The elements of a set can be anything, as long as they have a common characteristic. For
example, it can be a set of numbers, a set of books, a set of cows, a set of people, etc. To
this end, sets are then given names: in most cases sets are given capital letters as their names,
A, B, C, X, Y , etc. Intuitively1 , lets consider equal sets as those sets that have the same elements.
i.e. X = Y if set X has the same elements as set Y .
There are different ways of representing/describing a set and they are as follow;
A set can be represented by simply listing all its elements, separated by a comma (,).
e.g. A = {1, 2, 3, 4}. The arrangement/order of elements within the set does not matter,
so set A above is the same set as B = {3, 1, 2, 4}. i.e A = B. Elements of a given set
are unique within that set, so a repeated element in a set will just be counted once. e.g.
C = {1, 2, 3, 1, 4} is the same set as set A above.
A set can also be represented by giving a description of the (characteristics of the) objects
belonging to that particular set. e.g. D = all the positive integers less than 5. Note that
D contain exactly the same elements as A above.
A set can also be represented by using a mathematical notation called the set builder
notation. e.g. E = {x|x Z and 1 x 4}, which again has exactly the same elements
as A above.

Example 1.1.1 Determine whether the following statements are true or false. Give reasons.
a) 3 {1, 2, 3, 4}
b) 9 {x|x N and x 10}
c) {2, 4, 6, 8} =
6 {x|x is even and 0 < x < 10}

This is not the formal definition of the equality of sets. The formal definition will come later in the text.

Solutions
a) True, because 3 is really an element of {1, 2, 3, 4}.
b) False. Although 9 is a natural number, 9 < 10.
c) False. Because these two sets contain exactly the same elements.


1.2

Cardinality of Sets

When you are dealing with a collection of objects, its only natural that there is an interest
in the size of that particular collection, for several reasons. One is that smaller collections are
easier to deal with than larger collections. For instance, a Grade 12 class with 20 students is
easier to teach than that of say, 35 students. Therefore in mathematics we have a concept of
the cardinality of a set (which gives the size of that particular set).
Definition 1.2.1 (Set cardinality) Let X be a set. The cardinality of X, denoted by |X|
or sometimes n(X), is defined as the number of the elements of set X. i.e. |X| = n means that
set X contains n elements.

Definition 1.2.2 A set X is said to be finite (countable) if its elements can be counted (i.e.
the cardinality is a finite natural number). Otherwise the set is said to be infinite.

Remark 1.2.1 There exists a set with no elements called the empty set or the null set and
it is denoted by {} or .

Example 1.2.1 Find the cardinality of the following sets.


a) X = {4, 6, 8, 10, 12}
b) Y = {x|x N}
c) C =
d) A = {}

Solutions
a) |X| = 5.
b) Sice we can not count the elements of the set of all the natural numbers, we say that Y is
infinite or |Y | = .
c) Since C is an empty set and the empty set does not have elements, then |C| = 0.
d) |A| = 1 because A has only one element, namely .

Definition 1.2.3 If two sets have the same cardinality, then they are said to be equipotent
(equivalent). If sets A and B are equipotent we denote it by A B, otherwise A  B.
Example 1.2.2 If A = {10, 12, 14}, B = {a, b, c} and C = {}, then A B, but A  C because
|A| = 3, |B| = 3 and |C| = 1.

This is where we will leave the concept of equipotent sets (at least for now). If you continue
with mathematics after first year, then you will get to explore this concept more in advanced
courses on set theory.

1.3

Subset and Proper subset

Consider the collection of all the science students at the University of Namibia (UNAM). Now
this collection/group includes all the UNAM students majoring in mathematics among others.
i.e. The collection of all the mathematics majors at UNAM is a subcollection of the collection of
all the UNAM science students because every mathematics major student at UNAM is a UNAM
science student. To this end, we define a subset of a given set as follows;
Definition 1.3.1 Let X and Y be two sets. Set X is said to be a subset of set Y , denoted by
X Y , if and only if every element of set X is also an element of set Y . i.e. if x X then
x Y . One needs to note here that X and Y could be the same set. If X Y and X 6= Y then
we say X is a proper subset of Y and we denote this by X Y or X $ Y . One can then deduce
that, set X is said to be equal to set Y , denoted by X = Y , if and only if X Y and Y X.
Note that for two sets X and Y , X $ Y (X a proper subset of Y ) implies that, for all x X,
x Y , and there exist some y Y such that y
/ X. To show that set X is not a subset of set
Y , we use the notation X * Y .
5

Remark 1.3.1 Let X be any set. Then,


a) X X.
b) X.

Consider the following example.

Example 1.3.1 Let A = {5, 3, 7} and B = {1, 3, 5, 7, 9}.


a) Is A a subset of B?
b) Is B a subset of A?

Solution
a) Yes, A B because all the elements of A are also elements of B.
b) No, B * A because not all the elements of B are also elements of A, e.g. 1, 9 B but
1, 9
/ A.

Lets look at another example.

Example 1.3.2 Consider the following sets; A = {4, 6, 8, 10, 12, 14}, B = {12, 9, 10, 8, 14, 6, 16, 4, 18},
C = {4, 6, 8} and D = {12}. Determine whether the following statements are true of false.
a) A B
b) A $ B
c) D C
d) C B
e) A * C

Solutions

a) True. All the elements of A are contained in B (although not in the same order).
6

b) True. A B for the same reason given in a), and there are some elements of B that are not
elements of A. e.g. 16.
c) False. The only element of D (12) is not an element of C.
d) True. All the elements of C are contained in B.
e) True. There are some elements of A that are not elements of C. Come up with an example.


Remark 1.3.2 Let A, B, and C be sets.


1. If A B, then |A| |B|.
2. If A $ B, then |A| < |B|.
3. If A = B, then |A| = |B|.
4. If A B and B C, then A C and hence |A| |C|.

From Remark 1.3.1 it follows that for any given set X, we can find at least one subset of X
(more than one subset if X is nonempty, and exactly one if X = ). Therefore, for any given
set X, we are able to write down a set, whose elements are all the possible subsets of X, and we
define such a set as follows;
Definition 1.3.2 Let X be a set. The powerset of X, denoted by P(X) is the set whose elements
are all the possible subsets of X. The cardinality of the powerset of X is given by |P(X)| = 2|X| .

Example 1.3.3 Let X = {1, 2}. Then |X| = 2 and |P(X)| = 2|X| = 22 = 4.
Hence, P(X) = {, {1, 2}, {1}, {2}}.

In Example 1.3.3, |P(X)| = 4 means that X = {1, 2} has four subsets.
In most cases when you are considering a problem, the set of all the elements under considerations
is called the universal set and it is denoted by U. Therefore, all the sets you are considering are
actually subsets of some much larger set, U.

Exercises 1.1
1. Describe (i) in words, and (ii) using the set boulder notation, the sets whose elements are
listed below.
a) X = {3, 9, 27, 81, . . .}
b) Y = {5, 10, 15, 20, 25, . . . , 100}
2. List the elements, and give the cardinalities of the following sets.
a) A = The set of the powers of 6 with not more than 5 digits.
b) B = The set of three digit positive numbers whose digits are all the same.
c) C = {x N|x > 12} .
d) D = {x|x = n2 , n N}.
3. Given that X = {0, 1, 3, 7}, Y = {0} and Z = {0, 1, 2, 3, 4, 5, 6}. Determine whether each
of the following is true or false.
a) Y X
b) X Y
c) X Z
d) Y Z
4. Let X be a set defined by X = {x N|x3 < 50, x 6= 0}.
a) List the elements of X.
b) Find |P(X)|.
c) Find P(X).

1.4

Operations on Sets

One can perform operations on sets (within a specified U) in a similar manner as one can
perform operations, such as addition and multiplication, on numbers. We now define some basic
operations on sets.
Definition 1.4.1 Let X and Y be two sets.
a) The union of set X and set Y , denoted by X Y , is a set of all the elements that are either
in set X or in set Y or in both.
b) The intersection of set X and set Y , denoted by X Y , is a set of all the elements that
are in set X and in set Y . If the intersection of set X and set Y is an empty set, then we
say that sets X and Y are disjoint.
c) The complement of set X relative to set Y , denoted by Y \ X or Y X, is the set of all the
elements of set Y that are not in set X.
d) The complement of set X, denoted by X 0 or X c , is the set of all the elements of the universal
set U that are not elements of set X.

Remark 1.4.1 Also note the following:


(i) If X Y then X Y = X and X Y = Y .
(ii) X (Y Z) = (X Y ) Z and X (Y Z) = (X Y ) Z.
(iii) (X 0 )0 = X.

Study the following examples.

Example 1.4.1 Let A = {6, 8, 10, 12} and B = {3, 6, 9, 12}


a) Find A B.
b) Find A B.
c) Find A \ B and B \ A.

Solutions
a) A B = {6, 12}.
b) A B = {3, 6, 8, 9, 10, 12}.
c) A \ B = {8, 10} and B \ A = {3, 9}.

Theorem 1.4.2 Let X and Y be subsets of the universal set U. Then de Morgans laws for sets
state that
(i) (X Y )0 = X 0 Y 0 ,
(ii) (X Y )0 = X 0 Y 0 .

Example 1.4.2 Let U = {x Z|2 x 12}, X = {x Z|x is a prime number} and Y =


{x Z|x is a factor of 12}.
a) Find the following sets.
(i) X 0
(ii) Y 0
(iii) X Y 0
(iv) (X Y )0
b) Find the following values.
(i) |X|
(ii) |X 0 Y 0 |
Solutions
a) (i) X 0 = {4, 6, 8, 9, 10, 12}
(ii) Y 0 = {5, 7, 8, 9, 10, 11}
(iii) X Y 0 = {2, 3, 5, 7, 11} {5, 7, 8, 9, 10, 11} = {5, 7, 11}
(iv) (X Y )0 = X 0 Y 0 = {4, 6, 8, 9, 10, 12} {5, 7, 8, 9, 10, 11} = {4, 5, 6, 7, 8, 9, 10, 11, 12}
b) (i) |X| = 5
(ii) |X 0 Y 0 | = 9


10

1.5

Venn Diagrams

We use Venn diagrams2 to illustrate Sets and set operations diagrammatically. Venn diagrams
are made up of a square/rectangle, which represents the universal set, and circles inside the
square/rectange, which represent the sets under considerations. All the sets inside a universal
set are subsets of it. For instance, consider the following scenario:
Let U = {1, 2, 3, 4, 5, 6}, X be the set of all factors of 6: X = {1, 2, 3, 6}, and Y be the set of all
factors of 4: Y = {1, 2, 4}. Then we can represent this using a Venn diagram as follows:
'
U
5

X '$
Y
'$
3

&%
&%
&

Now, in the above Venn diagram, X Y (which is the set of the natural numbers that are
factors of both 4 and 6) is represented by the overlapping region of the two sets, and X Y is
represented by the region within the collective boundaries of X and Y .
We can also use Venn diagrams to indicate the number of elements in each set within the
universal set. In terms of the number of elements in the indicated set, the above scenario is
represented by a venn diagram as follows;
$

'
U

X '$
Y
'$
2

&%
&%

&

In the above diagram, the numbers appearing in each set are not elements of those particular
sets, but rather they indicate the number of elements belonging to those sets. For instance,
|X Y | = 2, |Y \ X| = 1 and so on.
Example 1.5.1 In a group of 7 science students, 3 major in mathematics, 4 major in physics,
and 2 do not major in mathematics and they do not major in physics.
a) Represent this scenario using a Venn diagram.
2

This method was invented by John Venn in 1880.

11

b) How many students in this group major in both mathematics and physics?
c) How many students major in maths but not in physics?

Solutions
a) Denote the set of the students majoring in mathematics with M and the set of the students
majoring in physics with P . Now, let x be the number of students that are majoring in
both mathematics and physics, i.e. |M P | = x. Then we have the following diagram;
'
U

M '$
P
'$
3x x 4x

&%
&%

&

b) Since the total number of students in the group is 7, from the venn diagram in a) we have
that
3x+x+4x+2 = 7
9x = 7
x = 2
c) The number of students that major in mathematics but not in physics is the cardinality of
the compliment of set P relative to set M , i.e. |M \ P |. So,
|M \ P | = 3 x
= 32
= 1


1.6

Intervals

Before we give a definition of, and discuss intervals, it would be most helpful if we give a brief
review of the number system.

1.6.1

Real Numbers

We all first encountered numbers when we learned counting. So, the numbers we learned first
are the whole (counting) numbers 0, 1, 2, 3, 4, . . . i.e. the set {0, 1, 2, 3, 4, . . .}. This set is called
12

the set of natural numbers, denoted by N. If one is only interested in the positive natural
numbers (without the 0) then we denote the set of all the positive natural numbers by N+ =
N \ {0} = {1, 2, 3, 4, . . .}. So, we have N+ N.
From there we moved onto fractions. For instance, one needed to represent dividing a whole
bread among 3 or 4 people equally. After that, we were introduced to assigning a negative sign
to positive natural numbers, such as 2, 7, and together with the natural numbers we obtain
the set of integers, denoted by Z = {. . . , 3, 2, 1, 0, 1, 2, 3, . . .}. Note that N Z.
Now, if we do the same to the positive fractions to obtain negative fractions, the set of integers
together with the set of positive and negative fractions give us the set of what is called the
rational numbers, denoted by Q. So, rational numbers are those numbers that can be represented as fractions of two integers. The reason integers are also rational numbers is that they
3
3
are fractions of themselves and 1. e.g. 5 = 15 = 5
1 , 3 = 1 = 1 . Again, note that Z Q.
Furthermore, there are numbers that cannot be represented as fractions of two integers. For

instance, 5, , etc. These are called the irrational numbers. The difference between rational
and irrational numbers is that rational numbers have terminating or repeating decimals, while
on the other hand, irrational numbers have nonterminating and nonrepeating decimals. Now, if
we put the rational and irrational numbers together, we get a set of what is called as the real
numbers, denoted by R. Again, we have Q R. The above story can be represented using a
Venn diagram as follows;
'

R
'$
Q

Z
Nj


&%
&

The real numbers can be represented on a number line, by choosing a point, marking it 0. Then
the positive real numbers are marked uniformly on the right side of 0, while the negative real
numbers are marked uniformly on the left side of 0. For example;

2 21

13

1
2

7
3

Theorem 1.6.1 (The Density of Real Numbers Property) Between any two rational numbers you can always find another rational number, and between any two irrational numbers you
can find another irrational number.
Theorem 1.6.1 implies that between any two real numbers there are infinitely many real numbers.

1.6.2

Intervals

For a given subset of the set of real numbers to be called an interval, it need to satisfy the
conditions given in following definition.
Definition 1.6.1 Let I R, and a, b I such that a < b. Then I is called an interval if given
any real number c such that a < c < b, we have c I.
You can think of intervals as corresponding to line segments;
I
I is an interval

- I is not an interval

Example 1.6.1
a) A = {a R| 1 a 1} is an interval because for any x, y A and c R such that
x < c < y, c A.
v

1 21 0

v
1
2

3
2

b) B = {1, 0, 1} is not an interval because for example 1, 1 B, and


but 21
/ B.

1
2

R where 1 <

1
2

<1

c) R is an interval, but N is not an interval.




14

We shall use the following notations for different types of intervals. For a, b R such that a b,
we have the following;
(a, b) = {x R|a < x < b}
f

[a, b] = {x R|a x b}
v

[a, b) = {x R|a x < b}


v

(a, b] = {x R|a < x b}


f

[a, +) = {x R|a x}
v

(a, +) = {x R|a < x}


f

15

(, b] = {x R|x b}
v

(, b) = {x R|x < b}
f

(, +) = R

1.6.3

Operations on Intervals

Intervals are sets, and therefore we can perform operations on them the same way as we did in
Section 1.4 on other sets.
For example, consider the two intervals A = (, 1) and B = (1, ). The intersection
of A and B is the set of all the real numbers that are less than 1 and grater than 1. i.e.
A B = (, 1) (1, ) = (1, 1). This can also be illustrated using the number line as
follows;
f
f

f
f

For the intervals A = (, 1) and B = (1, ), their union is the set of all the real numbers
that are either less than 1 or greater than 1. i.e. AB = (, 1)(1, ). This is illustrated
using the number line as follows;

16

f
f

f
f

17

Exercises 1.2
1. Let A = {2, 4, 8, 16, 32}, B = {4, 8, 12, 16, 20, 24, 24, 32}, and C = {3, 6, 9, 12, 15, 18, 21, 24, 27, 30}.
List the elements of the following sets.
a) A B
c) A B
e) B \ (A B)

b) A C
d) C (A B)

2. Two sets X and Y are such that |X| = 30, |Y | = 24, |X Y | = 47, and |U| = 50.
a) Represent this information using a Venn diagram.
b) Find the following:
(i) |X Y |
(ii) |X \ Y |
(iii) |Y \ X|
3. Determine whether each of the following sets is an interval and give sufficient reasoning.
a) R0
c) [3, 8] (5, 6)

b) Z
d) (9, 9)

4. Determine whether each of the following statements is true or false.


a) 0 (7, 0)
c) 13 (0, 31 ]
e) 5
/ (3, 6) \ {4, 5}

1.7

b) 8.7 [8, 9]
d) 4 (4, 6)

Summary

1. {} is not the same as , and is not the same as {0}.


2. The order of the elements in a set does not matter and theres no need to repeat elements
in a set. i.e X = {1, 3, 5, 7} = {3, 5, 7, 1} = {1, 7, 5, 3} and Y = {1, 3, 1} = {1, 3}.
3. All the elements of the powerset are sets.

18

1.8

Answers to Exercises in Unit 1

Exercise 1.1
1. (i) a) X = The set of all the powers of 3.
b) Y = The set of all the positive multiples of 5 not greater than 100.
(ii) a) X = {x|x = 3n , n N+ }
b) Y = {y|y = 5k, k N+ , y 100}
2. a) A = {6, 36, 216, 1296, 7776, 46656} and |A| = 6
b) B = {111, 222, 333, 444, 555, 666, 777, 888, 999} and |B| = 9
c) C = {13, 14, 15, 16, 17, . . .} and |C| =
d) D = {0, 1, 4, 9, 16, 25, . . .} and |D| =
3.

a) True
c) False

b) False
d) True

4. a) X = {1, 2, 3}
b) |P(X)| = 8
c) {, X, {1}, {2}, {3}, {1, 2}, {1, 3}, {2, 3}}

Exercise 1.2
1.
a) A B = {2, 4, 8, 12, 16, 20, 24, 28, 32}
c) A B = {4, 8, 16, 32}
e) B \ (A B) = {12, 20, 24, 28}

b) A C = {2, 3, 4, 6, 8, 9, 12, 15, 16, 18, 21, 24, 27, 30, 32}
d) C (A B) = {12, 24}

2. a)
'
U

X '$
Y
'$
30 y y 24 y

&%
&%

&

19

b) (i) |X Y | = 7
(ii) |X \ Y | = 23
(iii) |Y \ X| = 17
3.
a) not an interval
c) interval

b) not an interval
d) not an interval

4.
a) False
c) True
e) True

b) True
d) False

20

Unit 2

Algebraic Expressions
Content
2.1 Simplification and Expansion of Algebraic Expressions
2.1.1 Addition and Subtraction
2.1.2 Multiplication and Division
2.1.3 Factoring
2.1.4 Factoring Quadratic Expressions
2.2 Rational Exponents and Radicals
2.2.1 Radicals
2.2.2 Rationalization of Denominator and Numerator

Introduction
Most of algebra and advance mathematics are made up mainly of algebraic expressions. Knowing
how to deal with these expressions is thus very important. In this Unit, we are going to learn
about what constitute algebraic expressions, what like terms in an algebraic expression are and
how they can be dealt with. There is also a section on how to simplify and add algebraic
expressions.

Objectives
By the end of this Unit, you should be able to:
identify algebraic terms.

21

collect, add or subtract like terms.


multiply and divide algebraic expressions.
use the laws of exponents appropriately and well.
simplify rational expressions.
factorize all algebraic expressions.
rationalize numerators or denominators of fractional expressions involving indices.

Activities and Study Times


All the activities and studying this Unit should not take you more than eight (8) hours.

22

The ability to manipulate algebraic expressions correctly is an essential skill for advanced mathematics. But before we look at some of the basic algebraic manipulation, we need to know just
what algebraic expressions are.

Definition 2.0.1 Algebraic expressions are made up of variables (normally represented by letters), numerical constants and signs of operations.

For instance, 2x + y is an algebraic expression, where x and y are the variables (or variable
factors), 2 and 1 are the numerical constants, and addition and multiplication are the signs
of operations. Such an expression represents one quantity. Just as the sum of 1 and 2 is one
quantity, namely, 3, the sum of x and y is one quantity, namely, x + y.
Other examples of algebraic expressions are as follows:
a) 3x2 2x + y
b) ax2 + bx + c
ax
+y
b

d) 4 x + 3 3 x x
c)

When there is nothing between two variables (i.e. when there is no sign between two variables),
it is taken to mean that those two variables are being multiplied by one another. e.g. ax = a x,
and xyz = x y z. The numerical constants put in front of variables are called the coefficients
of those respective variables. For instance, for the algebraic expression in a) above, 3 is the
coefficient for x2 , 2 is the coefficient for x and 1 is the coefficient for y. Note therefore that,
when theres no numerical constant in front of a variable, the coefficient of that particular
variable is taken to be 1.
Theres another concept about algebraic expression that we need to define, and thats the concept
of terms of algebraic expressions. The terms of an algebraic expression are the parts of the
expression that are connected by plus or minus signs. For example, in the expression 3abx+cyk,
the terms are 3abx, cy, and k. An expression containing only one term such as 2x4 , is called a
monomial, while the ones that contain two terms are called binomials and the one that contain
three terms are called trinomials.

23

2.1

Simplification and Expansion of Algebraic Expressions

In order to be able to operate on algebraic expressions, there are fundamental properties that
you should be able to follow. The operations include addition/subtraction, multiplication, factorizing, using laws of exponents and to cancel like terms in the numerator and denominators
where necessary.
Definition 2.1.1 (Like Terms) Terms containing variables can be combined only if their variables parts are the same. Terms containing variable factors in which the same letters are raised
to the same power are called like terms.

For example, 5x and 9x are like terms because they have the same variable factor, namely x.
Also, 3a2 b and a2 b are like terms and the variable factor is a2 b.

2.1.1

Addition and Subtraction

Algebraic expressions are added or subtracted by adding or subtracting like terms. Like terms are
added or subtracted by adding or subtracting the constant (numerical) coefficients and placing
the result in front of the variable factor. For example, 5x + 9x = (5 + 9)x = 14x.
You are expected to know the basic sign manipulations. i.e. Things like xy = x+(y) = y+x,
or (x + y) = x y, and so on. You are also expected to know that multiplying a positive
with a negative number gives a negative answer, multiplying a positive number with a positive
number gives a positive number, and so does multiplying a negative number with a negative
number.
Example 2.1.1 Simplify the following expressions.
a) 3x + 7x
b) 2ab + 5ab
c) (5x y) (2x + 3y)
d) 4z (2 + 3z)
e) x + 4xy 2x + y y 2

Solutions

a) Since 3x and 7x are like terms, we can add them to have


3x + 7x = (3 + 7)x = 10x.
24

b) The two terms 2ab and 5ab are like terms. So,
2ab + 5ab = (2 + 5)ab = 7ab.
c) In this expression we first need to open the brackets, and then collect the like terms. The
minus sign in front of the second set of brackets means that you should multiply everything
inside the brackets with 1. If we do this we get
(5x y) (2x + 3y) = 5x y 2x 3y.
Now that we have opened the brackets, we need to collect the like terms, i.e. write the
like terms together and add/subtract them. Therefore,
5x y 2x 3y = 5x 2x y 3y = (5 2)x + (1 3)y = 3x 4y.
This implies that (5x y) (2x + 3y) = 3x 4y.
Now, since 3x and 4y are not like terms, we cant add them and this last expression is
as simple as it can get and therefore it is our answer.
d) We open the brackets and add/subtract the like terms.
4z (2 + 3z) = 4z 2 3z = (4 3)z 2 = z 2
Since z and 2 are not like terms, we cant add/subtract them and therefore this is our
answer.
e) We collect and add/subtract the like terms. Note that the only like terms in this expression
are x and 2x. So,
x + 4xy 2x + y y 2 = x 2x + 4xy + y y 2 = x + 4xy + y y 2 .
Now, since x, 4xy, y and y 2 are not like terms, we cant simplify this last expression
any further, and therefore it is our answer.


Addition/Subtraction of Fractional Algebraic Expressions


We do addition and subtraction of fractional algebraic expressions in a similar way as addition
5
and subtraction of fractional numbers. Remember that to add 38 to 12
, you need to first look for
the least common denominator (lcd ) (which is just the least common multiple of the denominators), make it the denominator for your answer. You then divide this lcd by the denominators
and multiply the respective numerators with the quotients and then add/subtract the products

25

(numerators). i.e.
3
5
+
8 12

=
=
=

33+25
24
9 + 10
24
19
24

(24 is the lcd )

In a similar way, we add/subtract fractional algebraic expressions as follows.


Example 2.1.2 Simplify the following algebraic expressions.
x s
a)
+
y
t
b) x
c)

2
2x + 1

1
3

x+1 x1

Solutions
a) We note that for these two algebraic fractions, the lcd is yt. Hence we have
x s
+
y
t

=
=

xt sy
+
yt
yt
xt + sy
yt

b) For this example, first we need to realize that x is also a fractional expression because it can
be written as x1 . The lcd therefore is just 2x + 1. Hence we have the following,
x

2
3x + 1

=
=
=

x
2

1 2x + 1
x(2x + 1)
2

2x + 1
2x + 1
2x2 + x 2
2x + 1

c) We note that the lcd here is (x + 1)(x 1) = x2 1. So,


1
3

x+1 x1

=
=
=

x1
3(x + 1)
2
2
x 1
x 1
x 1 (3x + 3)
x2 1
2x 4
x2 1
26

Its very important to know how to add/subtract these types of expressions


because youll need this knowledge later in the course when we get to discuss
partial fractions.


2.1.2

Multiplication and Division

Unlike in the case of addition and subtraction, we can multiply/divide two algebraic terms even
when they are not like terms. e.g. 2x y = 2xy, and 2xy x = 2xy
x = 2y. Before we take a look
at the laws and properties that will help us multiply/divide algebraic expressions with ease, we
briefly need to understand a few concepts.
Consider the monomial x2 . In this monomial, x is called the base, and 2 is called the exponent
(the power) which indicates how many times one needs to multiply x with itself. e.g. x2 = x x.
Now, if one is multiplying monomials like the one above, say x2 and x3 , we have
x2 x3 = (x x) (x x x) = x x x x x = x5 = x2+3
This means that if you are multiplying monomials which have the same base, then you just need
to keep the base and add the exponents. Similarly, for other operations we have the following
laws of exponents.
Theorem 2.1.1 (Laws of Exponents) Let x, y R and m, n Z. Then;
a) xm xn = xm+n
b)

xm
xn

= xm xn = xmn

c) (xm )n = xmn
d) (xy)m = xm y m ; (xm y)n = xmn y n
e) ( xy )m =

xm
ym ;

(y 6= 0)

f ) ( xy )m = ( xy )m ; Also ( x1 )n = xn ; xm =
g)

xm
y n

yn
xm ;

1
xm ;

(x, y 6= 0)

(x, y 6= 0)

h) If x 6= 0, then x0 = 1.
It is very important that you verify and convince yourself that the laws of exponents listed in
Theorem 2.1.1 above are indeed true. It does not help memorizing them if you dont understand
them.

27

Example 2.1.3 Simplify the following expressions.


a) (2x)(3x4 )
b) (2x23 )(5x79 )
c) (3s)(st)
d) 2x2 (x + 3) 5x(x2 + 3x)
e) x(x 1) x(2 x) 3(x + 5) 2x2

Solutions

a) For this expression, we multiply the coefficients and then we use the laws of exponents for
the variable factor.
(2x)(3x4 ) = (2 3)(x x4 )
= 6x1+4
= 6x5
b) We do the same as in a) to get,
(2x23 )(5x79 ) = (2 5)(x23 x79 )
= 10x23+79
= 10x102
c) For this expression, note that the two terms have factors that have the same base s. So, we
apply the laws of exponents for this one and then just multiply with the other factors. i.e.
(3s)(st) = 3 s1+1 t = 3s2 t.
d) For expressions like this one, you need to open the brackets first, and then collect and
add/subtract the like terms.
2x2 (x + 3) 5x(x2 + 3x) = 2x3 + 6x2 5x3 15x2
= (2 5)x3 + (6 15)x2
= 3x3 9x2

28

e) In this case, we do the same as in d).


x(x 1) x(2 x) 3(x + 5) 2x2 = x2 x 2x + x2 3x 15 2x2
= (1 2 3)x + (1 + 1 2)x2 15
= 6x + 0x2 15
= 6x 15

We divide algebraic expression in a similar way as multiplication, except that we should note
that x1 = x1 . Therefore, when we divide x with y, we are really just multiplying x with y 1 .
i.e.
x
= x y 1
y
Example 2.1.4 Simplify the following expressions.
a)

2x5
3x3

b)

xy 2 z
xz

c)

a2 bc3
ab1

Solutions

a) You just need to apply the laws of exponents. You are dividing, and therefore since the base
is the same, you keep the base, divide the coefficients, and and subtract the exponents.
2x5
3x3

= (2 3)x53
=

b)

c)

2 2
x
3

xy 2 z
xz

= x11 y 2 z 11

a2 bc3
ab1

= a21 b1(1) c3

= y2

= ab2 c3
29


Often we encounter cases where algebraic expressions of more than one term are being multiplied.
To multiply two algebraic expressions that have more than one term, multiply all the terms in
one expression with all the terms in the other expression, then simplify by adding/subtracting
the like terms if and where necessary. For example, (a + b)(c + d) = ac + ad + bc + bd
The following types of products occur frequently in mathematics, and hence we lay them down
explicitly. Let x and y represent any real numbers or algebraic expressions, then

a) (x + y)(x y) = x2 y 2

(Difference of squares)

b) (x + y)2 = x2 + 2xy + y 2

(Perfect square)

c) (x y)2 = x2 2xy + y 2

(Perfect square)

d) (x + y)3 = x3 + 3x2 y + 3xy 3 + y 3

(Perfect cube)

e) (x y)3 = x3 3x2 y + 3xy 2 y 3

(Perfect cube)

Lets study the following example.

Example 2.1.5 Multiply and simplify the following expressions.


a) (x + 1)(x + 2)
b) (3x + 7)(4x2 x + 3)
c) (2xy z)(z + 2)
d) (x2 x 3)(x2 + 3x 1)

Solutions

a) Remember, we multiply each term in the fist expression with each term in the second expression. i.e.
(x + 1)(x + 2) = x(x + 2) + 1(x + 2)
= x2 + 2x + x + 2
= x2 + 3x + 2
30

b)

(3x + 7)(4x2 x + 3) = 3x(4x2 x + 3) + 7(4x2 x + 3)


= 12x3 3x2 + 9x + 28x2 7x + 21
= 12x3 + 25x2 + 2x + 21

c)

(2xy z)(z + 2) = 2xy(z + 2) z(z + 2)


= 2xyz + 4xy z 2 2z

d)

(x2 x 3)(x2 + 3x 1) = x2 (x2 + 3x 1) x(x2 + 3x 1) 3(x2 + 3x 1)


= x4 + 3x3 x2 x3 3x2 + x 3x2 9x + 3
= x4 + 2x3 7x2 8x + 3


Multiplication/Division of Fractional Algebraic Expressions


To multiply two (or more) fractional algebraic expressions, you multiply the numerators with
one another, and then the denominators with one another. The (unsimplified) answer would
be a fraction with the numerator a product of the numerators of the two (or more) fractional
expressions, and the denominator a product of the denominators of the two (or more) fractional
expressions. Again, this method is similar to the method of multiplying fractional numbers.

Example 2.1.6 Multiply the following algebraic fractions and simplify.


a)

x s

y
t

b)

y
14x
2
21x4
y

c)

z+1
3

4
z + 12

Solutions

a)

x s
xs
=
y
t
ty

31

b)

y
14x
2
21x4
y

=
=

c)

14xy
21x4 y 2
2
3x3 y

z+1
3
3(z + 1)
3z + 3

=
=
4
z + 12
4(z + 12)
4z + 48


Division of fractional algebraic expressions is performed in an almost similar manner as multiplication, but you need to know the following definition.
Definition 2.1.2 Let x be an algebraic expression. The reciprocal of x is defined as

1
or x1 .
x

Now, dividing an algebraic expression with x is the same as multiplying that particular expression
y
with the reciprocal of x. If x is a fractional algebraic expression (i.e. x = ), then the reciprocal
z
1
1
y
z
of x is
= y = . Therefore, dividing an algebraic expression with
is the same as
x
(z)
y
z
z
multiplying it with . Let us thus consider the following examples:
y
Example 2.1.7 Simplify the following expressions.
x s

a)
y
t
b)

y
14x
2
4
21x
y

c)

z+1
3

4
z + 12

Solutions
a) Note that the reciprocal of

s
t
is . Therefore we have
t
s

x s
x t
xt
= =
y
t
y
s
ys
b) The reciprocal of
y
14x
2
4
21x
y

=
=

14x
y2
is
. Hence
y2
14x
y
y2

21x4 14x
y3
294x5
32

c) The reciprocal of

3
z + 12
is
. Hence
z + 12
3

z+1
3

4
z + 12

=
=
=

z + 1 z + 12

4
3
(z + 1)(z + 12)
12
z 2 + 13z + 12
12


Exercises 2.1
1. Simplify the following.
a) 3x + 9x 2x
b) 4xy + y + 7xy
c) 8x (4x + 2)

d) 7 x 3 x + 2 x
3 2
e) +
y y
1
3
f)
y 4y + 1
8
2
+
g)
x 1 3x 1
2
1
h)
+
x 2 (x 2)2
2. Simplify the following.
a) (4xy)(xy)(7xy)


1
2
b) (3xy)(2x )
3xy
254 42
1253 161
 2 3 2
3x y
d)
y 2 x1
e) (2x 5)(x + 7)
c)

33

2.1.3

Factoring

Before we look at how an algebraic expression can be factored, we will briefly give a statement
on what factoring actually is.
Remember from your elementary number theory that if a, b, and c are integers different from
zero such that c = ab, then a and b are called factors of c. We will give an informal definition,
in a similar way, of what a factor is with reference to algebraic expressions.
Definition 2.1.3 Let x, y, and z be algebraic expressions. If z = xy then x and y are said to
be factors of z. The process of writing z as a product of its factors is called factoring z (or
sometimes factorizing z).

For example, the algebraic expression 3x2 + 9x has factors 3, x, and x + 3 since it can be written
as 3x2 + 9x = 3x(x + 3).

Common Factors
The first thing you need to know about factoring is the concept of the greatest common factor
(gcf ). The greatest common factor (gcf ) is a greatest number or letter which is a factor of all
the terms in the algebraic expression. For example, for the expression 3x2 + 9x, the gcf for
the terms 3x2 and 9x is 3x. Now, factorizing an algebraic expression involves dividing every
term in the expression by the gcf and writing the expression as a product of the gcf and the
sum/difference of the resultant quotients. For instance, for the expression 3x2 + 9x, we said that
the gcf is 3x, and therefore after we divide every term in the expression with 3x and writing
the expression as a product of 3x with the sum of the quotients we get 3x2 + 9x = 3x(x + 3).
Note that sometimes not all the terms in an expression have a common factor, but one may still
be able to do some factoring. Here are some examples for you to follow:
Example 2.1.8 Factorize completely the following algebraic expressions.
a) 6x3 y 9x2 y 3 + 12xy
b) 10a2 + 5a + 2ab + b
c) x2 + 2xy + 5x3 + 10x2 y

34

Solutions
a) Note that the gcf in the expression 6x3 y 9x2 y 3 + 12xy is 3xy. Therefore we have
6x3 y 9x2 y 3 + 12xy = 3xy(2x2 ) + 3xy(3xy 2 ) + 3xy(4)
= 3xy(2x2 + 3xy 2 + 4)

dividing every term by the gcf

multiplying the gcf by the sum of the quotients

b) For the expression 10a2 + 5a + 2ab + b not all the terms have a common factor, but we can
see that the first two terms have a common factor which is 5a and the last two terms have
a common factor which is b. Therefore we get the following
10a2 + 5a + 2ab + b = 5a(2a + 1) + b(2a + 1)

now we have a common factor x = 2a + 1, i.e.

= 5ax + bx
= x(5a + b)

factoring x out

= (2a + 1)(5a + b)
c) We have a gcf x. So,
x2 + 2xy + 5x3 + 10x2 y = x(x + 2y + 5x2 + 10xy)
= x(1(x + 2y) + 5x(x + 2y))
= x(1 + 5x)(x + 2y)

factoring inside the brackets

factoring x + 2y out of the brackets




2.1.4

Factoring Quadratic expressions

Quadratic expressions are algebraic expressions with the general form


ax2 + bx + c,
where a, b, c R. Now, not all quadratic expressions can be factorized, but those that can be
factorized must have two linear factors, i.e factors of the form dx + e for d, e R.
Factoring x2 + bx + c by Trial and Error
The process of finding the gcf sometimes is really a matter of trial and error. We are starting
with a quadrating expressions in which a = 1.
Consider the expression x2 + bx + c. To factorize this expression, we need to find two factors of
c whose sum is b. i.e. We needs to find two real numbers d and e (by trial and error) such that
35

c = de and b = d + e. This implies also that bx = dx + ex. We then rewrite the expression in
the following form;
x2 + bx + c = x2 + dx + ex + de
= x(x + d) + e(x + d)
= (x + e)(x + d)

since bx = dx + ex and c = de
factoring as in the previous subsection

factoring x + d out

Hence, we have managed to write the quadratic expression x2 + bx + c as a product of its two
linear factors. One should see therefore that the main task really is finding the two factors of c,
whose sum is b, and the rest is employing things we covered in the previous subsection.
Example 2.1.9 Factorize the following quadratic expressions.
a) x2 + 5x + 6
b) x2 + x 2
c) x2 3x 4
Solutions
a) The factors of 6 are 1, 2, 3, and 6 itself. Now, the two factors of 6 whose sum is 5 are 2 and
3. Therefore we have,
x2 + 5x + 6 = x2 + 2x + 3x + 2 3
= x(x + 2) + 3(x + 2)
= (x + 2)(x + 3)
b) The factors of 2 are 1, 1, 2, and 2 itself. The two factors of 2 whose sum is 1 are 1
and 2. So,
x2 + x 2 = x2 x + 2x 2
= x(x 1) + 2(x 1)
= (x 1)(x + 2)
c) The factors of 4 are 1, 2, 1, 2, 4, and 4 itself. The two factors of 4 whose sums is 3
are 1 and 4. So,
x2 3x 4 = x2 + x 4x 4
= x(x + 1) 4(x + 1)
= (x + 1)(x 4)
36

Factoring ax2 + bx + c by Trial and Error


We are looking at quadratic expressions where a 6= 1. For such expressions, we should expect
factors of the form ix + j and kx + l, where i, j, k, l R. i.e
ax2 + bx + c = (ix + j)(kx + l)
2

= ikx + (il + jk)x + lj

(2.1.4.1)
(2.1.4.2)

Now, from equation 2.1.4.2 we have that ac = iklj and the numbers il and jk (whose sum is b)
are factors of ac. Therefore to factorize the quadratic expression ax2 + bx + c, we need to find
two real numbers il and jk such that they are factors of ac and their sum is b.
Example 2.1.10 Factorize the following quadratic expressions.
a) 3x2 + 5x + 2
b) 2x2 + 11x + 12
c) 8x2 + 10x 3

Solutions

a) The factors of 3 2 = 6 are 1, 2, 3 and 6. The two factors whose sum is 5 and whose product
is 6 are 2 and 3. So,
3x2 + 5x + 2 = 3x2 + 3x + 2x + 2
= 3x(x + 1) + 2(x + 1)
= (x + 1)(3x + 2)
b) The factors of 2 12 = 24 are 1, 2, 3, 4, 6, 8, 12 and 24. The two factors whose sum is 11 and
whose product is 24 are 3 and 8. So,
2x2 + 11x + 12 = 2x2 + 8x + 3x + 12
= 2x(x + 4) + 3(x + 4)
= (x + 4)(2x + 3)

37

c) The factors of 8 (3) = 24 are 12, 8, 6, 4, 3, 2, 1, 1, 2, 3, 4, 6, 8, 12, 24 and 24.


The two factors of 24 whose sum is 10 and product is 24 are 2 and 12. So,
8x2 + 10x 3 = 8x2 2x + 12x 3
= 2x(4x 1) + 3(4x 1)
= (4x 1)(2x + 3)

The following are some special factoring formulae. Let x and y be numbers or algebraic terms,
then
a) x2 y 2 = (x y)(x + y)

(Difference of squares)

b) x2 + 2xy + y 2 = (x + y)2

(Perfect square)

c) x2 2xy + y 2 = (x y)2

(Perfect square)

d) x3 y 3 = (x y)(x2 + xy + y 2 )

(Perfect cube)

e) x3 + y 3 = (x + y)(x2 xy + y 2 )

(Perfect cube)

Remark 2.1.2 In a) above, expressions x y and x + y are said to be conjugates of each


other.

2.2

Rational Exponents and Radicals

When we looked at the exponents in Theorem 2.1.1, the expression xn was defined for x a real
number and n an integer. In this section we want to modify the assumptions of Theorem 2.1.1
to take care of cases where n is any rational number. e.g. we want to look at the meaning of
1
expressions like 3 2 , and write down laws of dealing with such expressions. Therefore we have
the following definition.
Definition 2.2.1
1

If n is an even positive integer and x 0, then x n is the nonnegative real number such
1
that (x n )n = x.
1

If n is an odd positive integer, then x n is the real number such that (x n )n = x.

38

For all positive integers m and n such that


1
numbers x for which x n is a real number,
m

m
n

is in its simplest form, and for all real

x n = (x n )m = (xm ) n .

For instance, 9 2 = 3 because 32 = 9; (64) 3 = 4 because (4)3 = 64; and 9 2 = (9 2 )3 =


3
1
1
33 = 27, or 9 2 = (93 ) 2 = 729 2 = 27.
1

Remark 2.2.1 Note that if n is an even positive integer and x < 0, the x n is not a real number.

In the same way that we have laws of exponents when the exponent is an integer (Theorem
2.1.1), we have laws of exponents when the exponent is a rational number. The following laws
should be followed when one is working with rational exponents.

Theorem 2.2.2 (Laws of Rational Exponents) Let n and m be rational numbers, and x
and y be positive real numbers. Then
a) xn xm = xn+m
b)

xn
= xnm
xm

c) (xn )m = xnm
d) (xy)n = xn y n
 n
x
xn
e)
= n
y
y
f ) xn =

1
xn

Example 2.2.1 Simplify the following expressions.


1

a) (81x12 y 16 ) 4
b)

x2 y 6
x2 y 6


c)

x2 y 4
x2 y

 12

39

Solutions
Note that an exponential expression is in its simplest form when no powers of powers or negative
exponents appear.
a) We use the laws of rational exponents in Theorem 2.2.2 to simplify the expression as follows.
1

= 81 4 (x12 ) 4 (y 16 ) 4

(81x12 y 16 ) 4

12

= 3x 4 y

16
4

= 3x3 y 4
b) In this expression we use the fact that
1

x2 y 6

1
b

= b1 . So,

 3 1 1
1 5
= x2 y 6 x2 y 6

x2 y 6

= (x 2 x 2 )(y 6 y 6 )
1

= x22 y 66
4

= x1 y 6
2

y3
x

c) Simplifying this we get,




x2 y 4
x2 y

 12

(x2 y 4 ) 2
1

(x2 y) 2
1

=
=

(x2 ) 2 (y 4 ) 2
1

(x2 ) 2 y 2
xy 2
1

x1 y 2
1

= x1(1) y 2 2
3

= x2 y 2

2.2.1

Radicals

When you are working with rational exponents, you are bound to encounter expressions of the

form n x. We will briefly look at how such expressions are defined and how they are handled.

Definition 2.2.2 In the expression n x, the square root symbol is called the radical sign, x
is called the radicand, and the positive integer n is called the index of the radical.
40

Now, the relationship between rational exponents and radicals is that; if n is a positive integer
1
and x is any real number such that x n is defined as a real number then

x = xn .

1
1
1
For example, 3 64 = 64 3 = 4, and 4 81 = 81 4 = 3, but 4 16 = (16) 4 is not defined as a real
number (according to Remark 2.2.1).
If the index of the radical is 2 then the radical is written without specifying the index, i.e.

2
x = x, which is referred to as the square root of x. x is the positive square root x while

x is the negative square root of x. e.g. 4 = 2 and 4 = 2.


From above, it then follows that if n is a positive integer, m any integer, and x any real number

such that n x is defined as a real number, then

m
1
x n = (x n )m = ( n x)m = n xm .

Example 2.2.2 Evaluate the following expressions.

a) ( 3 8)2

b) ( 3)2

Solutions

2
1
a) ( 3 8)2 = 8 3 = (8 3 )2 = 22 = 4

2
b) ( 3)2 = 3 2 = 3


1
Combining the fact that n x = x n and the laws in Theorem 2.2.2 we have the following laws,
which should be followed when one is dealing with radicals.

Theorem 2.2.3 (Laws of Radicals) Let n and m be integers greater than or equal to 2, and
let x and y be nonnegative real numbers. Then

a) n x n y = n xy

r
n
x
x
b)
= n
(y 6= 0)
n y
y
41

c)

x=

mn

d) ( n x)n = x

e) n xn = x

Simplification of algebraic expressions involving radicals is in line with these rules.

Example 2.2.3 Simplify the following expressions.

a) 81x2
p
b)
50x9 y 11
p
c) 3 16x4 y 5

Solutions
Note that a radical expression is said to be simplified (at least so far) if the following are met;
The powers of the radicand are less than the index.
The index of the radical is as small as possible.
The radicand is not a fraction.

p
81x2 = (9)2 x2 = 92 x2 = 9x
p
p
p

b)
50x9 y 11 = (25x8 y 10 )(2xy) = (2x4 y 5 )2 2xy = 5x4 y 5 2xy
p
p
p
p
p
c) 3 16x4 y 5 = 3 (8x3 y 3 )(2xy 2 ) = 3 (2xy)3 3 2xy 2 = 2xy 3 2xy 2
a)

2.2.2

Rationalization of Denominator and Numerator

Firstly, we need to remember that if you have a quantity and you multiply it with 1 you get
the same quantity. i.e. the value of ones quantity does not change when multiplied with 1.
For example 12 1 = 12 . Rationalizing denominator and numerator of an algebraic fraction
(involving radicals) is based on this fact.
Definition 2.2.3 Rationalization is a process of changing a radical denominator or numerator
to a perfect nth power without changing the value of the algebraic expression.
42

To rationalize, we multiply the algebraic fraction with an (radical) expression equivalent to 1,


that will make the appropriate radicand a perfect nth power.

Example 2.2.4 Rationalize the denominator in each of the following expressions.


1
a)
2
7
b)
3
a
Solutions

a) We need to find an expression equivalent


to 1 that will turn the radicand in the denominator

2
into a perfect square. So, we find 2 = 1 and therefore,

1
1
2
2
2
= = =
.
2
2
2
2
2
2
b) Going the similar route as in a) we get

3 2
3
3
7
7
a
7 a2
7 a2

.
=

=
=
3 3
3
3
a
a
a 3 a2
a

3 2
a

3 2
a

= 1, So,


The following example is on rationalizing the numerators of fractional expressions.

Example 2.2.5 Rationalize the numerator in the following expressions.

2
a)
3

5 3
b)
6
Solutions

a) We proceed in a similar manner as in Example 2.2.4, only we do it to the numerator. So,


2
2
2
22
2
=
= =
3
3
2
3 2
3 2


5 3
5 3
3
5 32
53
5
b)
=
= = = .
6
6
3
6 3
6 3
2 3
43


We look at even more complicated examples on rationalization.

Example 2.2.6 Rationalize the denominator in the following expressions.


a)

x+ y

b)

x+h

Solutions

a) We multiply the top and the bottom with the conjugate to x + y, namely x y to
get,

x y
4
4

x+ y
x+ y
x y

4( x y)
=

( x + y)( x y)

4( x y)
difference of squares
=
xy
b) The conjugate of our denominator is

x+h

=
=
=
=

x+

x + h. So,

x+ x+h
3

x x+h
x+ x+h

3( x + x + h)

( x x + h)( x + x + h)

3( x + x + h)
difference of squares
x (x + h)

3( x + x + h)
h


44

Exercises 2.2
1. Factorize the following expressions completely.
a) a3 b3 b3 .
b) 18x4 y 2 + 12x3 y 3 24x2 y
c) 4x2 2xy 6x + 3y
d) 3x2 2ax 3bx + 2ab
2. Factorize the following expressions
a) x2 + 6x + 5
b) 9x2 + 10x + 1
c) 22x2 77x + 33
d) 3x2 5xy + 2y 2
3. Find all the values of k such that the trinomial x2 + kx + 16 is a perfect square trinomial.
4. Simplify the following expressions completely.
3

a) (81x12 y 20 ) 4

8
8
b) 27 x3 29 x13
p
12x3 y 5
c) p
20x4 y
5. Rationalize the numerator/denominator of the following expressions.
p
3 6 x2 y 3
a)
7x 6
3
b)

5+ x

x+h x
c)
h

2.3

Summary

1. Always factor out the greatest common factor of all terms.


2. After each factorization, examine the new factors to see whether they can be factored.

45

2.4

Answers to Exercises in Unit 2

Exercise 2.1
1. a) 10x
b) 11xy + y
c) 4x 2

d) 6 x
5
e)
y
y+1
f)
y(4y + 1)
26x 10
g)
(x 1)(3x 1)
x
h)
(x 2)2
2. a) 28x3 y 3
b) 2x2
c) 5
d) 9x6 y 2
e) 2x2 + 9x 35

Exercise 2.2
1. a) b3 (a3 1)
b) 6x2 y(3x2 y 2xy 2 4)
c) (2x 3)(2x y)
d) (x b)(3x 2a)
2. a) (x + 1)(x + 5)
b) (9x + 1)(x + 1)
c) 11(2x 1)(x 3)
d) (3x 2y)(x y)
3. k = 8 or k = 8.

46

1
27x9 y 15
b) 4x2
y2
c)
70x
10x

4. a)

3x2 y 3
p
(7x 6) 6 (x2 y 3 )5

3( 5 x)
b)
5x
1
c)

x+h+ x

5. a)

47

Unit 3

Equations
Content
3.1 Linear Equations
3.2 Simultaneous Linear Equations
3.3 Quadratic Equations
3.3.1 Solving by Factoring
3.3.2 Solving by Completing the Squares
3.3.3 Solving by Using the Quadratic Formula
3.4 Simultaneous Nonlinear Equations

Introduction
Most real life situations can be modeled using equations (linear or nonlinear). So, knowing how
to solve equation is an essential tool, not only in mathematics, but also in life in general. In
this Unit, we will look at methods of solving linear and nonlinear equations in one variable, and
methods of solving linear and nonlinear simultaneous equations in two variables.

Objectives
By the end of this Unit, you should be able to:
solve single linear equations.
solve linear simultaneous equations.
48

determine whether a quadratic equation has real solutions, and the type of solutions.
solve quadratic equations.
solve nonlinear simultaneous equations.

Activities and Study Times


All the activities and studying this Unit should not take you more than seven (7) hours.

49

An equation is a statement that states that two mathematical expressions are equal. For example,
(i) 4 + 5 = 9
(ii) 7x 1 = 12 + x
The above statements are equations. Example (ii) contains an unknown variable, and what one
wants is to find the value of the unknown variable that would make the equation true. This
value is called the solution of the equation, and the process of finding a solution to an equation
is called solving the equation.
Equations with the same solutions are called equivalent equations and we use the symbol to
indicate that two equations are equivalent. Now, if you are given an equation, you can obtain
an equivalent equation by performing the same mathematical operation (addition, subtraction,
multiplication, division, etc.) on both the left and the right hand side of the equation. Here are
some properties of equity: Let the letters A, B, C stand for any algebraic expressions,
a) A = B A C = B C
b) A = B CA = CB (C 6= 0)

Remark 3.0.1 Note that, whatever operation you perform on one side of the equal sign, you
should perform the same operation on the other side as well to obtain an equivalent equation.

Before we look at ways of solving equations, we say some brief statements about a very important
concept in solving equations, Inverse operations.
Definition 3.0.1 (Inverse Operations) Two operations are called inverse operations if one
reverses the effect of the other.

For example, addition and subtraction are inverse operations. Multiplication and addition are
also inverse operations. This is to say that, if you start with an algebraic expression x, and you
add 2 to x, you obtain x + 2. Now, if you then subtract 2 from this new expression x + 2, you
obtain (x+2)2 = x+(22) = x+0 = x. So, you obtain an expression x which you started with.
On the other hand, if you start with the same expression x and you multiply with 2, you obtain
2
2x. Now if you divide this new expression with 2, you obtain 2x 2 = 2x
2 = 2 x = 1 x = x,
which is again what you started with.
50

3.1

Linear Equations

Definition 3.1.1 A linear equation in one variable is an equation of the form ax + b = 0, where
a and b are real numbers and x is the variable.

We solve linear equations by using inverse operations to rewrite the given equation into an
equivalent equation of the form
variable = number.
Consider the following example.

Example 3.1.1 Solve the following equations


a) 2x + 7 = 31
b) 4x + 7 = 9x 3
Solutions
a) Remember, we want to obtain an equivalent equation with the variable x on one side, and
everything else on the other side, by performing appropriate inverse operations on both
sides of the equal sign, as follows.
2x + 7 = 31
(2x + 7) 7 = 31 7
2x = 24
1
1
2x =
24
2
2
x = 12

subtract 7 from both sides to undo the effect of +7 on the left

divide both sides by 2 to have x on its own on the left.

If you want to confirm that the value that you obtained is indeed the solution to your
equation, just substitute that value for your variable in the equation, and you should obtain
the same values on both sides of the equal sign. i.e. LHS1 = 212+7 = 24+7 = 31 =RHS2 ,
which is what you have on the right hand.

1
2

LHS stands for Left Hand Side


RHS stands for Right Hand Side

51

b) Note that this is also a linear equation because, by using the inverse operations, we can
rewrite this equation into an equivalent equation of the form ax + b = 0. i.e. we collect
the like terms.
4x + 7 = 9x 3
(4x + 7) 7 = (9x 3) 7

subtract 7 from both sides

4x = 9x 10
4x 9x = (9x 10) 9x
5x = 10
1
1
( ) (5x) = ( ) (10)
5
5
x = 2

subtract 9x from both sides

divide both sides by 5 to have x on its own


From Example 3.1.1 b) we saw that there are some linear equation which are not of the form
ax + b = 0, but which can be rewritten into an equivalent equation of the form ax + b = 0. The
following are some of such examples.

Example 3.1.2 Solve the following equations.


a)

x2
2
=
x+1
3

b)

3
5
=
2x + 3
x1

Solutions

a) We use inverse operations to write this equation into a linear form and solve.
x2
x+1

2
3
2
x2 =
(x + 1)
3
3(x 2) = 2(x + 1)
=

3x 6 = 2x + 2
3x 2x = 2 + 6

multiply both sides by x + 1


multiply both sides by 3
expand both sides
collect like terms

x = 8

52

b) We follow the same procedure as in a) above to obtain the following.


5
3
=
2x + 3
x1
5(x 1) = 3(2x + 3)
5x 5 = 6x + 9
5 9 = 6x 5x

multiply both sides by (x 1)(2x + 3)


expand both sides
collect like terms

14 = x
x = 14

Equations in Example 3.1.2 can also be referred to as the rational equations. In general, only
some of the rational equations have equivalent equations in linear form. We will look at those
that do not have equivalent equations in linear forms later in the course.

3.2

Simultaneous Linear Equations

All the equations that we have considered so far only contain one variable. For instance 2x+7 = 3
is an equation in one variable only, namely x. There are however cases/systems were the number
of variables involved is more than one.
Now, for the system to have a unique solution (i.e. exactly one value for each variable) the
number of equations representing the system must be the same as the number of variables. For
example, the equation x + y = 2 is an equation in two variables namely, x and y, and the
values x = 1 and y = 1 form a solution to this equation because 1 + 1 = 2. But you should
also note that the values x = 4 and y = 2 also form a solution to this same equation since
4 + (2) = 4 2 = 2.
This means that this equation does not have a unique solution. To have a unique solution, we
need to introduce a second equation in two variables, and then we solve this pair of equation
together. This will give the values of x and y that satisfy both the two equations. Solving
more than one equation together is referred to as solving simultaneous equations, or some times,
solving the equations simultaneously.
If these equations are all of the form ax + by + c = 0 (for equations in two variables) then we
call them simultaneous linear equations. In this course, we will only restrict ourselves to solving
two equations in two variables simultaneously.

53

There are several methods of solving equations simultaneously. We will discuss two of them
here.
Substitution Method
To solve simultaneous equations by the substitution method, you make one of the variables the
subject of the formula3 in one equation, and then substitute it in the other equation. This gives
you an equation in one variable, which we now know how to solve.
Example 3.2.1 Solve the equations simultaneously in each case.

a)

2x y = 2
x+y = 7

b)

4x + y
= 6
2x + 3y = 24

Solutions

a) So, we consider this pair of equation and try to solve it using the substitution method.
2x y = 2
x+y = 7
If we make y the subject of the formula in the second equation we get y = 7 x. Now, in
place of y in the first equation, we substitute with 7 x and solve for x to get;
2x (7 x) = 2
2x 7 + x = 2
3x = 2 + 7
9
x =
3
x = 3
Now, since y = 7 x, we have y = 7 3 = 4.
Note that you can verify whether the solution youve obtained is correct by substituting
the values of x and y into the equations and see if the equations are satisfied.
3

Making a variable the subject of the formula means that you use the inverse operations to have only that
variable on one side of the equal sign and everything else on the other side.

54

b) We follow the same procedure here as we did in a).


4x + y = 6
2x + 3y = 24
This time we are making y the subject of the formula in the first equation to get y =
6 4x. We substitute this into the second equation and solve to get;
2x + 3(6 4x) = 24
2x 18 12x = 24
14x = 24 + 18
42
x =
14
x = 3
y = 6 4x = 6 4(3) = 6 + 12 = 6.


The Addition/Subtraction Method


This method works best when one of the variables has either the same coefficient in both equations or its coefficient in one equation is minus its coefficient in the other equation. That way,
when you add the two equations, that variable gets eliminated and you obtain one equation in
one variable, which we know how to solve.
If you have a pair of equations where no variable has the same coefficient in both equation and
no variable has a coefficient in one equation which is minus its coefficient in the other equation,
then you will need to multiply one of the equations with an appropriate real number to obtain
one of the two cases. We will illustrate this in the following examples.
Example 3.2.2 Solve the following pairs of equations simultaneously.

a)

x + 3y = 11
x+y = 3

b)

4x + y = 5
3x 2y = 1

55

Solutions

a) So we have the pair of equations


x + 3y = 11
x+y = 3
And we note that the variable x has the same coefficient, 1, in both equations. We then
proceed by subtracting the second equation from the first one as follows:
x + 3y = 11
x+y = 3
2y = 8
y = 4
Now, having obtained the value of y, we can substitute it in any of the two equation to
find the value of x. If we, for instance, decide to substitute it into the first equation, we
get the following;
x + 3 4 = 11
x + 12 = 11
x = 1
b) In this case we have the following pair of equations.
4x + y = 5
3x 2y = 1
Now, since none of the variables have the same coefficient is both the equations, and not
one has a coefficient in one equation that is minus its coefficient in the other equation,
we try multiplying one of the equation with an appropriate real number. Note that if we
multiply the first equation with 2, the coefficient of y is the first equation would be 2 and
in the second equation is already 2, which is what we want. So, we get,
2(4x + y = 5)
3x 2y = 1
Which works out to be
8x + 2y = 10
3x 2y = 1
56

Now we add the second equation to the first one to get the following;

8x + 2y = 10
3x 2y = 1
11x = 11
x = 1
Substituting this value of x in the first equation we get,
41+y = 5
4+y = 5
y = 1

You should get the same solution for a given pair of simultaneous equation regardless of which
method you use. The choice as to which method to use depends on your personal preference
(and maybe on the one you think saves you some time). Unless the method is specified, you are
welcome to use any method you are comfortable with, as long as you apply it the right way and
obtain the correct answer.
We now turn our attention to nonlinear equations. Although there are other nonlinear equations
that are well worth our time, in this course we will restrict ourselves to the quadratic equations.

Exercises 3.1
1. Solve the following equations.
a) 2x + 3 = x 4
2x + 1
b)
=0
3x 5
14
2 5
c) + =
y y
9
2
6
=
d)
x+1
4x 3
2. Use the substitution method to solve the following equations simultaneously.
2x y = 1
3x y = 2
57

3. Use the addition/subtraction method to solve the following equations simultaneously.


3x + 2y = 2
2x 3y = 16

3.3

Quadratic Equations

Definition 3.3.1 A quadratic equation is an equation of the form ax2 + bx + c = 0, where a, b, c


are real numbers with a 6= 0.

While all the quadratic equations have solutions, only some of the quadratic equations have real
solutions. In this course, if a quadratic equation does not have real solutions we would say that
it does not have a solution4 . So, before we start solving quadratic equations, it makes sense
to first know which quadratic equations have solutions (and what type of solution) and which
quadratic equations do not have solutions.

The Discriminant
Definition 3.3.2 Let a, b and c be real numbers such that a 6= 0. Then the discriminant of the
quadratic equation ax2 + bx + c = 0, denoted by 4, is given by
4 = b2 4ac.

Now, for a given quadratic equation, the discriminant determines if it has real solutions and the
type of solutions it would have, as follows:
(i) If 4 > 0 then the quadratic equation ax2 + bx + c = 0 has two distinct real solutions.
(ii) If 4 = 0 then the quadratic equation ax2 + bx + c = 0 has one (repeated) real solution.
(iii) If 4 < 0 then the quadratic equation ax2 + bx + c = 0 has no real solution.

The reasons for the above three statements will become clear a little later, but for know, lets
just take them as given.
4

As long as you keep in mind that it does have non-real solutions.

58

Example 3.3.1 Determine the number of real solutions the following quadratic equations have.
a) 2x2 3x + 1 = 0
b) x2 14x + 49 = 0
c) 2x2 + 5x + 7 = 0
Solutions
We use the discriminant to determine the number of solutions for each equation.
a) Note that for this equation a = 2, b = 3 and c = 1. So,
4 = b2 4ac
= (3)2 4 2 1
= 98
= 1
Since 4 > 0, the equation has two distinct real solutions.
b) In this case a = 1, b = 14 and c = 49. Therefore,
4 = b2 4ac
= (14)2 4 1 49
= 196 196
= 0
That is, 4 = 0 and hence the equation only has one (repeated) real solution.
c) For this equation a = 2, b = 5 and c = 7. So,
4 = b2 4ac
= (5)2 4 2 7
= 25 56
= 31
Now, since 4 < 0, the equation does not have real solutions.

In this course, we will look at three ways of solving quadratic equations, for those that have real
solutions. The first one is, solving by factoring.

59

3.3.1

Solving By Factoring

Before we go any further, we need to state a very important tool in solving equations by factoring,
namely The Zero-Product Property.
Theorem 3.3.1 (The Zero-Product Property) Let and be algebraic expressions. Then
= 0

if and only if

= 0 or = 0.

Now, lets consider the quadratic equation ax2 + bx + c = 0. The left hand side of this equation
is just a quadratic expression. The methods and procedures of factoring all kinds of quadratic
expressions that can be factored are outlined in Section 2.1.4.
We factor the left side of our quadratic equation and get an equation of the following form;
(ix + j)(kx + l) = 0.
Applying Theorem 3.3.1 to our resultant equation we get that
ix + j = 0

or

kx + l = 0.

Solving these two linear equations we get the two solutions x = ji or x = kl . So, lets consider
the following example.

Example 3.3.2 Solve the following quadratic equations using the factoring method.
a) x2 + 3x 10 = 0
b) 6x2 + 21x 12 = 0

Solutions

a) We factor the left-hand side of the equation x2 + 3x 10 = 0 and use Theorem 3.3.1 to solve
for x.
x2 + 3x 10 = 0
(x + 5)(x 2) = 0

factoring the left-hand side of the equation

x + 5 = 0 or x 2 = 0

using Theorem 3.3.1

x = 5 or x = 2
So, the solutions for our equation are x = 5 and x = 2.
60

b) Similarly, we factor the left-hand side of the equation 6x2 + 21x 12 = 0 and solve to have,
6x2 + 21x 12 = 0
(3x + 12)(2x 1) = 0

factoring the left-hand side of the equation

3x + 12 = 0 or 2x 1 = 0
1
x = 4 or x =
2

using Theorem 3.3.1

So, the solution for our equation are x = 4 and x = 12 .




3.3.2

Solving by Completing the Squares

The whole idea of completing the squares is to obtain a perfect square after factoring. Consider
the expression x2 + bx. To obtain a perfect square from this expression we need to add a third
2
term. For example for the expression x2 + 6x, we need to add the term 9, which is 12 6 to
get x2 + 6x + 9. Now, factoring this we get (x + 3)2 . Note that 3 is half of 6. This implies that
2
2
for the expression x2 + bx we need to add 2b to get x2 + bx + 2b , and when we factor this
we get (x + 2b )2 . This is the idea we use in completing the squares.
So, for the quadratic equation of the form ax2 + bx + c = 0, where a 6= 0, we first factor a
out of the first two terms to get a(x2 + ab x) + c = 0, and subtract c from both sides to get
a(x2 + ab x) = c. Now completing the squares on the expression inside the brackets on the
left-hand side we get,


b
a x2 + x
= c
a
 2 !
b
b2
b
= c +
whatever you do on the left, do it on the right as well.
a x2 + x +
a
2a
4a


b2
b 2
=
c factoring inside the brackets on the left.
a x+
2a
4a


b 2
b2 4ac
a x+
=
2a
4a

2
2
b
b 4ac
x+
=
2a
4a2

After obtaining the equation above, we then use the following fact to solve for x.
Theorem 3.3.2 The solutions of the equation x2 = c are x =
61

c and x = c.

Lets therefore consider the following examples.

Example 3.3.3 Solve the following quadratic equations by completing the squares.
a) x2 + 6x + 1 = 0
b) 2x2 + 4x 1 = 0

Solutions

a) We start by taking 1 to the right-hand side, and then proceed to complete the squares on
the left-hand side as follows.
x2 + 6x + 1 = 0
x2 + 6x = 1
x2 + 6x + 9 = 1 + 9

adding 9 on both sides

(x + 3)2 = 8

x+3 = 8
x = 3

factorizing the left-hand side

taking the square root on both sides


8

Therefore, the solutions to our equation are x = 3 +

8 and x = 3

8.

b) As in a) above, we start by taking 1 to the right-hand side and then complete the squares
on the left-hand side.
2x2 + 4x 1 = 0
2x2 + 4x = 1
1
x2 + 2x =
2
1
2
x + 2x + 1 =
+1
2
3
(x + 1)2 =
2r
3
x+1 =
2

dividing throughout by 2
adding 1 on both sides
factoring the left-hand side
taking the square root on both sides
r

x = 1

3
2

Therefore, the solutions to our quadratic equation are x = 1 +

62

3
2

and x = 1

3
2.

3.3.3

Solving by using The Quadratic Formula

For the quadratic equation ax2 + bx + c = 0, the formula for obtaining its solutions, referred to
as the quadratic formula, is obtained through the method of completing the squares. We will
then state it as a theorem, and prove it.
Theorem 3.3.3 (The Quadratic Formula) The solutions of the quadratic equation ax2 +
bx + c = 0, where a 6= 0, are

b b2 4ac
x1,2 =
.
2a

Proof: Consider the quadratic equation ax2 + bx + c = 0, where a 6= 0. Completing the squares
on this equation we get,


b
2
a x + x
= c
a
!
 2
b
b2
b
2
= c +
whatever you do on the left, do it on the right.
a x + x+
a
2a
4a


b 2
b2
a x+
=
c factoring inside the brackets on the left.
2a
4a


b2 4ac
b 2
=
a x+
2a
4a
2

2
b 4ac
b
=
x+
2a
4a2
r
b
b2 4ac
=
taking the square root on both sides
x+
2
2a
4a

b
b2 4ac
x1,2 =
2a
2a
2
b b 4ac
x1,2 =
.
2a
As required.

If you are required to use the quadratic formula, you dont need to prove it again. Instead, just
identify your a, b and cs and substitute them in the formula to get your solutions. You however
need to understand the proof because you can definitely be asked to reproduce it during the
evaluations. We will then try to use the quadratic formula in the following example.

63

Example 3.3.4 Use the quadratic formula to solve the following quadratic equations.
a) x2 + x 1 = 0
b) 2x2 + 4x 1 = 0

Solutions

a) Note that for our equation, a = 1, b = 1 and c = 1. So, we substitute these values into the
quadratic formula to get,

b b2 4ac
x1,2 =
2a
p
1 12 4 1 (1)
=
21
1 5
=
2
1
5
=
2
2
So, the solutions are x = 21 +

5
2

and x = 12

5
2 .

b) Here, a = 2, b = 4 and c = 1. So,

b b2 4ac
x1,2 =
2a
p
4 42 4 2 (1)
=
22
4 24
=
4
6
= 1
2

So, the solutions are x = 1 +

6
2

and x = 1

6
2 .

Remark 3.3.4 From Theorem 3.3.3 it should be apparent now why and how the discriminant
4 = b2 4ac determines whether the solutions for a given quadratic equation are real or not.

64

3.4

Simultaneous Nonlinear Equations

Before we conclude the Unit on equations, we will take a brief look at simultaneous nonlinear
equations. In Section 3.2, we looked at ways of solving simultaneous linear equation, and some
(or maybe all) of those ways can also be used to solve simultaneous nonlinear equations.
Simultaneous nonlinear equations is a set of equations where one or more of the equations are
not linear equations. For example,
x+y = 2
xy = 1
are simultaneous nonlinear equations because the equation xy = 1 is not a linear equation.
Whether solutions for given simultaneous nonlinear equations exist or not, and the number of
equations, depends on the type of equations you are considering.
We will try to solve the simultaneous nonlinear equations above (and some others) using the
substitution method in the following example.
Example 3.4.1 Solve the following simultaneous nonlinear equations.
a)

x+y = 2
xy = 1

b)

3 + y = x2 + 2x
y = x+1

Solutions
a) For these equations, we make y the subject of the formula in the first equation to have,
y = 2 x. We then substitute this into the second equation for y, and obtain an equation
in x only, which we will solve as follows;
xy = 1
x(2 x) = 1

substituting with 2 x for y

2x x 1 = 0
x2 2x + 1 = 0

dividing though by 1

(x 1)2 = 0
x1 = 0
x = 1
65

Substituting this value of x into the equation y = 2 x we get that y = 2 1 = 1.


b) For these equations, we use the second equation to substitute for y into the first equation,
and then try to solve for x as follows;
y + 3 = x2 + 2x
(x + 1) + 3 = x2 + 2x
x + 4 = x2 + 2x
x2 + x 4 = 0

collecting the like terms

This is a quadratic equation in x, which we cant seem to factorize. So, we try the quadratic
formula.

b b2 4ac
x1,2 =
2a
1 1 + 16
=
2

1 17
=
2

So, we have x = 21 217 and x = 12 + 217 . Substituting these into


the equation

17
17
17
1
1
1
y = x + 1 we get, when x = 2 2 , y = 2 2 , and when x = 2 + 2 , y = 21 + 217 .

In the following example, we will try to solve simultaneous nonlinear equations using the addition/subtraction method.

66

Example 3.4.2 Solve the following simultaneous equations.

a)

y = 2x2 x + 1
y = x2 + 2x + 5

b)

2x2 + 3y 2 = 11
3x2 + 2y 2 = 19

Solutions
a) Since y has the same coefficient in both equations, we subtract the second equation from the
first one to get the following;
y

= 2x2 x + 1

= x2 + 2x + 5

0 = x2 3x 4
(x 4)(x + 1) = 0
x 4 = 0 or x + 1 = 0
x = 4 or x = 1
Substituting these values into one of the equations (say the first one) we get, when x = 4,
y = 2 42 4 + 1 = 29, and when x = 1, y = 2 (1)2 + 1 + 1 = 4.
b) Since in this case we dont have a variable with the same numerical coefficient in both
equations, we create it by multiplying through the first equation by 3 and thought the
second equation by 2, and proceed to try and solve as follows.
6x2 + 9y 2 = 33
6x2 + 4y 2 = 38
5y 2 = 5
y 2 = 1
Now, since the equation y 2 = 1 has no real solutions (4 = 4), the simultaneous
equations do not have real solutions as well.

This is where we leave it as far as equations are concerned. Solving equations is very important
in mathematics and its applications. If theres any type of equations which are not covered in
67

this Unit, its because it is believed that you should be able to work out their solutions using
one or more of the tools discussed in this Unit. From here onwards, it is assumed that you know
how to solve equations.
Exercises 3.2
1. Determine the discriminant of the following quadratic equations, and then classify the
solutions of the equation as (i) two distinct real numbers, (ii) one real number (repeated
real solutions), or (iii) no real solutions. You dont have to solve the equations.
a) x3 + 3x + 3 = 0
b) x2 20x + 100 = 0
c) x2 + 3x 11 = 0
2. Solve the following equations by factoring.
a) 3x2 7x = 0
b) x2 2x 15 = 0
3. Solve the following equations by completing the square.
a) 2x2 + 10x 3 = 0
b) 2 + 10x 5x2 = 0
c) x2 6x = 0
4. Solve the following equations by using the quadratic formula.
a) 2x2 + 4x 1 = 0
b) x2 5x 24 = 0
5. Solve the following equations simultaneously.
y = x2
y = x+2

3.5

Summary

1. Whatever you do to the one side of the equal sign, you should do to the other side as well.
2. The existence of real solutions to the quadratic equation ax2 + bx + c = 0 is determined
by the discriminant 4 = b2 4ac.

68

3.6

Answers to Exercises in Unit 3

Exercise 3.1
1. a) x = 7
1
b) x =
2
9
c) y =
2
d) x = 6
2. x = 3 and y = 7.
3. x = 2 and y = 4.

Exercise 3.2
1. a) 4 = 3, So no real solutions.
b) 4 = 0, So repeated real solutions.
c) 4 = 53, So distinct real solutions.
7
3
b) x = 3 or x = 5

3. a) x = 52 12 31 or x = 52 + 21 31

b) x = 1 15 35 or x = 1 + 15 35
2. a) x = 0 or x =

c) x = 0 or x = 6

4. a) x = 1 12 6 or x = 1 + 21 6
b) x = 6 or x = 1.
5. When x = 2, y = 4 and when x = 1, y = 1

69

Unit 4

Inequalities
Content
4.1 Linear Inequalities
4.2 Nonlinear Inequalities

Introduction
In this Unit, we will learn what each of the inequality signs mean, and we will look at methods
of solving inequalities. We will also learn how solutions to inequalities can be represented on a
number line.

Objectives
By the end of this Unit, you should be able to:
use the properties of inequalities.
solve linear inequalities.
solve nonlinear inequalities.
represent solutions of inequalities on a number line.

Activities and Study Times


All the activities and studying this Unit should not take you more than five (5) hours.
70

Its quite often that we hear people or ourselves making statements like: Life is not fair . This
is because weve all observed that, not all things in life are equal or at the same level. For
example, some people are taller than others, some are richer than others, some leave closer to
the cities/towns than others, etc. You can think of so many other similar statements.
The above statements portray lifes inequalities. Inequalities can also be represented mathematically, but before we give a somewhat vague definition of what an inequality is mathematically,
we need to first define inequality signs we will be using.
The following are called the inequality signs, and this is how they are read;
< is read less than.
> is read greater than.
is read less than or equal to (i.e < or =).
is read greater than or equal to (i.e > or =).

A statement like x > y is read x is greater than y, and it is true only if x is a number greater
than the number y. A statement like x y is read x is less than or equal to y, and it is true
when x is less than y and it is also true when x is equal to y. We then give the definition of an
inequality.

Definition 4.0.1 An inequality is an expression that takes a similar form to that of an equation,
but in place of an equal sign, one has one or more of the following inequality signs: <, >, , .

The following are examples of inequalities:


1. 2x + 9 1 + 5x
2. x + 1 < 6

The process of solving an inequality which has a variable involves finding the set of values of
the variable which satisfy that particular inequality. These sets of solutions are in most cases
intersections or unions of intervals. From Unit 1, weve seen that intervals can also be represented
on a number line. So, we can also represent solutions of inequalities on a number line.
For example, we can represent the inequality x < 2 as

71

While the inequality 1 x 1 can be represented as follows;


v

So, we use a little circle o for inequalities of the form x < a or x > a, to indicate that a is not
included in the interval, and we use a solid dot for inequalities of the form x a or x a to
indicate that a is included in the interval.
We now write down some properties of inequalities, which we will use to solve inequalities.

Theorem 4.0.1 (Properties of Inequalities) Let x, y and z be algebraic expressions. Then,


a) x y x z y z
b) If z > 0, then x y xz yz
c) If z < 0, then x y xz yz
d) If x, y > 0, then x y

1
x

1
y

e) If x y and , then x + y +

The above properties are based on the inverse operations, and the same thing we used in solving
equations, that, any operation you perform on one side of the inequality, you should perform it
on the other side as well.
We will briefly give a special mention with regard to properties c) and d) in theorem 4.0.1
above. Property c) says that if you divide or multiply throughout an inequality by a negative
real number then the inequality sign changes directions. For instance, consider the inequality
3 < 7. This is true because 3 is indeed less than 7. Now, if we multiply throughout this inequality
by, say, 1, on the left hand side of the inequality sign, we get 3, while on the right hand side
we get 7. If we keep the inequality sign as it is, we will get something like 3 < 7, which is
false. To make it true, we have to invert the inequality sign, from < to >, to read 3 > 7.

Property d) says that if you take the reciprocal on both sides of the inequality sign then the
inequality sign must change directions as well. For example, the inequality 3 7 is true, but
72

the inequality 13 71 is not true. To make it true, we invert the inequality sign to read
So, please keep these in mind when solving inequalities.

1
3

71 .

We will then try to solve inequalities using these properties, starting with the linear inequalities.

4.1

Linear Inequalities

Linear inequalities are inequalities whose terms are linear with respect to the variable. For
instance if a, b, c, , , , are constants, then ax + b c and x + > x are linear
inequalities with respect to the variable x, and so is the inequality ax + b < c.
Example 4.1.1 Solve the following inequalities.
a) 3x + 11 6x + 8
b) 1 < 3x + 4 16
Solutions
a) We use the inverse operations to try and have the variable x on its own as follows.
3x + 11 6x + 8
3x + 11 6x 6x + 8 6x
3x + 11 11 8 11

subtracting 6x from both sides

subtracting 11 from both sides

3x 3
1
1
( )(3x) ( )(3) dividing both sides by 3
3
3
x 1 note that the inequality sign has reversed
Therefore, the solution to this inequality is a set of all the real numbers greater than or
equal to 1. i.e. the interval [1, ). We can represent this solution on the number line as
v

- -

b) We follow a similar procedure as in a) to get


1 < 3x + 4 16
1 4 < 3x + 4 4 16 4

subtracting 4 throughout the inequality

3 < 3x 12
1 < x 4

dividing though out by 3


73

Therefore the solution is the following set, (1, 4]. We can represent this on a number line
as follows
f

4.2

Nonlinear Inequalities

These are inequalities whose terms are nonlinear with respect to the variable. The important
tool in finding solutions to these kinds of inequalities will be factorization. Before we start
solving nonlinear inequalities, we need to note the following two facts.
The inequality x y > 0 is true if, either x > 0 and y > 0, or x < 0 and y < 0.
The inequality x y < 0 is true if, either x < 0 and y > 0, or x > 0 and y < 0.

We will use these two facts in solving the inequalities.

Example 4.2.1 Solve the following inequalities


a) x2 3x 18 0
b) x2 + 5x + 6 > 0
c)

x1
x+2
<
x+3
x2

Solution

a) x2 3x 18 0
Factorizing the left hand side we get, (x 6)(x + 3) 0
Now, to find the set of all the real numbers that satisfy this inequality, there are two ways
we can do this. Either we use one of the two facts listed above, or we construct what is
referred to as a sign table.

74

Using the first method we have,


(x 6)(x + 3)

[x 6 0 and x + 3 0] or [x 6 0 and x + 3 0]
[x 6 and x 3] or [x 6 and x 3]
[x (, 6] and x [3, )] or [x [6, ) and x (, 3]]
x (, 6] [3, ) or x [6, ) (, 3]
x [3, 6] or x
x [3, 6]
x [3, 6]
This implies that the solutions to our inequality are all the real numbers in the interval
[3, 6].
We now try to solve the same inequality using the sign table method to see if we get the
same solution.
We first identify the zeroes of the quadratic expression (values of x that will make the
quadratic expression zero) on the left hand side, which are 3 and 6 and we use them as
the end points to our intervals. Therefore we have the following sign table:
Interval

(, 3)

(3, 6)

(6, )

Sign of (x 6)
Sign of (x + 3)

+
+

Sign of (x 6)(x + 3)

Now we have that (x 6)(x + 3) = 0 at 3 and 6, and from the sign table, we see that
(x 6)(x + 3) is negative in the interval (3, 6). Therefore, the solution of the inequality
(x 6)(x + 3) 0 is [3, 6], which is the same as the one we obtained using the first
method. So, the two methods give the same solutions.
From here on, we will use the sign table method in this text, but if you feel more comfortable
with the first method, then please use that.
b) x2 + 5x + 6 > 0. Factorizing the left hand side we get, (x + 2)(x + 3) > 0, whose zeroes are
2 and 3. The sign table is as follows:

75

Interval

(, 3)

(3, 2)

(2, )

Sign of (x + 2)
Sign of (x + 3)

+
+

Sign of (x + 2)(x + 3)

From the sign table we see that (x+2)(x+3) is positive in the interval (, 3) or (2, ).
Therefore, the solution of the inequality (x + 2)(x + 3) > 0 is (, 3) (2, ).
c)

x+2
x1
<
.
x+3
x2
We start by moving all the nonzero terms to one side of the inequality sign, and we get,

x+2
x+3
x+2 x1

x+3 x2
(2x + 1)
(x 3)(x 2)

<

x1
x2

< 0
< 0

subtracting the two fractions on the left

From this, we see that the zero for the numerator is 12 and the zeroes for the denominator
are 2 and 3, where the fraction on the left hand side of the inequality is not defined. We
then use these zeroes to construct our interval to get the following sign table:
Interval

(, 12 )

( 12 , 2)

(2, 3)

(3, )

Sign of (2x + 1)
Sign of (x 2)
Sign of (x 3)
Sign of (x 2)(x 3)
(2x + 1)
Sign of
(x 2)(x 3)

+
+
+

x+2
x1
<
is ( 12 , 2) (3, ). Note that
x+3
x2
(2x + 1)
2 and 3 are not included in this solution set because the fraction
is not
(x 2)(x 3)
defined at these values.
Therefore, the solution of the inequality

76

Exercises 4.1 Solve the following inequalities and represent the solutions on the number line.
a) 4(x 5) 2x + 15
b) 6 <
c)

6x + 1
< 13
4

x
0
3x 5

d) 12x2 + 8x 15
e)

(x + 1)(x 4)
<0
x2

4.3

Summary

1. When solving linear or quadratic inequalities, write the inequalities so that you have the
nonzero terms on one side of the inequality sign and the zero on the other. Factorize the
nonzero terms and use the sign table to determine the solution intervals.
2. Multiplying through an inequality with a negative number, or taking reciprocal on both
sides of the inequality reverses the inequality sign.

77

4.4

Answers to Exercises in Unit 4

Exercise 4.1


a)
b)
c)
d)


5
,
6


23 51
,
6 6


5
0,
3

 

3
5
, ,
2
6

e) (, 1) (2, 4)

78

Unit 5

Absolute Value
Content
5.1 Properties of Absolute Value
5.2 Equations Involving Absolute Values
5.3 Inequalities Involving Absolute Values

Introduction
In this Unit, we will learn the graphical and algebraic meaning of an absolute value of a real
number. We will look at properties of absolute value and methods of solving equations and
inequalities involving absolute values.

Objectives
By the end of this Unit, you should be able to:
find the absolute value of a real number.
Use properties of absolute value.
solve equations involving absolute values properly.
solve inequalities involving absolute values properly.

Activities and Study Times


All the activities and studying this Unit should not take you more than five (5) hours.
79

Lets first get an intuitive feel of what an absolute value of a number is. Intuitively, the absolute
value of a number can be taken as the distance from zero to that particular number. Consider
the following diagram.
2 units

2 units
-

From the above diagram, we see that the number 2 is 2 units from zero, and so is the number 2.
Therefore, from the statement we made above, it follows that the absolute value of 2 and 2 is
the same, namely 2. Now, from high school, youve learned that distance is always positive, and
therefore the absolute value of a number (positive or negative), except 0, is always a positive
number. The absolute value of 0 is just 0 because 0 is 0 units from zero.
We then give the formal algebraic meaning of an absolute value of a number in the following
definition.

Definition 5.0.1 Let x be a real number. Then the absolute value of x, denoted by |x|, is defined
as follows:
(
x
if x 0;
|x| =
x if x < 0.
For example, |2| = 2, and | 2| = (2) = 2, while |0| = 0, which agrees with our diagram
above.

Example 5.0.1 Write each of the following expressions without absolute value symbols.
a) | 8|
b) |4| + | 10| | 3|
c) |x + 6| + |x 2|, given that 0 < x < 1.




x+7
, given that 0 < x < 1.
d)
|x| + |x 1|
Solutions

a) We use the definition of absolute value. Since 8 < 0, we get


| 8| = (8) = 8.
80

b) Since 4 > 0, |4| = 4. On the other hand, 10, 3 < 0, so | 10| = (10) = 10 and
| 3| = (3) = 3.
|4|+|10||3| = 4+103 = 11.
c) Note that for 0 < x < 1, x + 6 > 0 while x 2 < 0. So, from the definition of absolute value
we have that |x + 6| = x + 6 and |x 2| = (x 2). Substituting these into our expression
we get,
|x+6|+|x2| = (x+6)+((x2)) = x+6x+2 = 8.
d) Firstly, we need to realize that for 0 < x < 1, |x| = x and x 1 < 0, which implies that
|x 1| = (x 1). This implies further that |x| + |x 1| = x x + 1 = 1, and



x + 7

x+7
=


|x| + |x 1| 1 = |x+7| = x+7
because x + 7 > 0 for 0 < x < 1.


5.1

Properties of Absolute Value

The absolute value expressions in Example 5.0.1 are relatively simple to simplify, and we havent
really used anything we dont already know for our discussion on algebra. For more complicated
expressions however, we need to state the properties of absolute value, which we use to simplify
and/or evaluate absolute value expressions.
Theorem 5.1.1 (Properties of Absolute Value) Let x and y be expressions, such that y 6=
0. Then,
1. |x| 0
2. |x| = | x|
3. |xy| = |x||y|
4. |x y| = |y x|

x |x|
5. =
y
|y|
6. |x|2 = x2

81

Remark 5.1.2 The first property in Theorem 5.1.1 means that the absolute value of any nonzero
real number is always positive and |0| = 0.

We will now see how we can use the properties in Theorem 5.1.1 to simplify absolute value
expressions in the following example.

Example 5.1.1 Simplify the following expressions.


a) |( 31 )(15)|

6
b)
24




x
+
3


c)
, given that 0 < x < 15 .
|x 21 | + |x + 12 |
Solutions

a) |( 31 )(15)| = | 13 || 15| =

6 | 6|
6
1
b) =
=
= .
24
|24|
24
4

1
3

15 = 5.

c) We write the absolute value of a quotient as a quotient of absolute values as follows,






x+3
|x + 3|


.

=
1
1
|x 2 | + |x + 2 | ||x 12 | + |x + 21 ||
Now, since 0 < x < 15 , |x + 3| = x + 3, |x 12 | = x + 12 , and |x + 12 | = x + 12 . If we
substitute these back we get,
x+3
x+3
x+3
|x + 3|
= x+3.
1
1 =
1
1 = |1| =
1
||x 2 | + |x + 2 ||
| x + 2 + x + 2|


82

5.2

Equations Involving Absolute Values

We need to consider the diagram we had at the beginning of the Unit.


2 units

2 units
-

From this diagram, we have |2| = 2 and | 2| = 2. Now if you are told that |x| = 2, then
there are only two possibilities, namely, either x = 2 or x = 2. We can extend this further
to say that; if you are told that |x + 3| = 2, then again there are only two possibilities: either
x + 3 = 2 or x + 3 = 2. This fact forms the basis for solving equations involving inequalities.
The equations x + 3 = 2 and x + 3 = 2 can be solved using the methods we discussed in Unit
3. Lets consider the following example.
Example 5.2.1 Solve the following equations for x.
a) |3x| = 12
b) |x 2| = 8
c) |4 3x| = 16

Solutions

a) We use the facts stated above and the methods discussed in Unit 3 to have,
|3x|

12

3x = 12 or 3x = 12
12
12
x=
or x =
3
3
x = 4 or x = 4
So, the solutions to our equation are x = 4 and x = 4.
b) Similarly, we have
|x 2|

x 2 = 8 or x 2 = 8
x = 8 + 2 or x = 8 + 2
x = 10 or x = 6
Therefore, the solutions are x = 10 and x = 6.
83

c) Solving this equation, we get


|4 3x|

16

4 3x = 16 or 4 3x = 16
3x = 16 4 = 12 or 3x = 16 4 = 20
12
20
x=
or x =
3
3
20
x = 4 or x =
3
Therefore, the solutions are x = 4 and x =

20
3 .


Note that an equation of the form |x + 3| = 8 do not have solutions because absolute value
of any number is nonnegative. So, since the right hand side of the equation is negative, the
equation would not have a solution.
In the last property in Theorem 5.1.1 we have seen that we can actually get rid of the absolute
value sign by squaring the expression. Lets apply that in the following example.

Example 5.2.2 Solve the following equations for x.


a) |x + 4| = |x|
b) 2|x 3| + 5|x 3| = 7x 21

Solutions

a) We proceed by squaring both sides of the equation to get rid of the absolute value sign, and
then solve.
|x + 4| = |x|
|x + 4|2 = |x|2
(x + 4)2 = x2

squaring both sides


applying the last property in Theorem 5.1.1

x2 + 8x + 16 = x2
8x + 16 = 0
8x = 16
x = 2

84

b) For this equation, first we simplify.


2|x 3| + 5|x 3| = 7x 21
7|x 3| = 7(x 3)
|x 3| = x 3
From the definition of absolute value, we know that this is only true if x 3 0, which
implies that x 3. So the solution to our equation is the interval [3, ).


5.3

Inequalities Involving Absolute Values

To see how inequalities involving absolute values are solved, we need to take another look at our
diagram.
2 units

2 units

We let x be a real number. Now, if x is between 2 and 0 then its distance from zero is less than
or equal to 2 units (i.e. |x| 2). If x is between 0 and 2 then we have the same situation, |x| 2.
Therefore we have |x| 2 if either x [2, 0] or x [0, 2] (i.e. x [2, 0] [0, 2] = [2, 2]).
On the other hand, if x is to the left of 2 then its distance from zero is greater than 2 units
(|x| > 2). We have the same situation, |x| > 2, if x is to the right of 2. Therefore we have
|x| > 2 if either x < 2 or x > 2 (i.e. x (, 2) (2, )).
In general, we have the following properties to apply when solving inequalities that involve
absolute values.
Theorem 5.3.1 Let x and a be real numbers such that a > 0. Then,
a) |x| < a a < x < a
b) |x| a a x a
c) |x| > a x < a or x > a
d) |x| a x a or x a
We use these properties to solve inequalities involving absolute values, as follows in this example.

85

Example 5.3.1 Solve the following inequalities


a) |x 5| 3
b) |x + 1| > 1
Solutions

a)

|x 5| 3 3 x 5 3
2x8
The solution set is therefore the interval [2, 8].

b)

|x + 1| > 1 x + 1 < 1 or x + 1 > 1


x < 2 or x > 0
Therefore the solution set is the interval (, 2) (0, ).


In Theorem 5.1.1 we saw that |xy| = |x||y| and | xy | = |x|


|y| . For addition however, its not always
the case that the absolute value of the sum of numbers is a sum of the absolute values of these
individual numbers. With regard to addition, we have the following theorem called the triangle
inequality.
Theorem 5.3.2 (Triangle Inequality) Let x and y be real numbers. Then
|x + y| |x| + |y|.
We use the triangle inequality to solve problems of the following type.
Example 5.3.2 Given that |x 4| < 0.2 and |y 7| < 0, 1, estimate |(x + y) 11|.
Solution
|(x + y) 11| = |(x + y) (4 + 7)|
= |(x 4) + (y 7)|
|x 4| + |y 7|

from Triangle Inequality

< 0.2 + 0.1

given

= 0.3
Therefore |(x + y) 11| < 0.3.


86

Exercises 5.1
1. Simplify the following expressions.
a) | 2| + | 3| | 5|




x+7
, given that 0 < x < 1.

b)
|x| + |x 1|
2. Solve the following equations for x.
a) |x + 14| = 20
b) 3|x 5| 16 = 2


x + 3

=6
c)
4
3. Solve the following inequalities
a) |2x 9| < 7
b) 4 |2x 2| 2
4. Given that |x| < 0.6 and |y + 1| < 0.18, use the triangle inequality to estimate |x 1 + y|,
clearly indicating where you are applying the triangle inequality.

5.4

Summary

1. The absolute value of a real number is always positive.


2. Your knowledge on equations and inequalities is also needed in solving equations and
inequalities involving absolute values.

87

5.5

Answers to Exercises in Unit 5

Exercise 5.1
1. a) 0
b) x + 7
2. a) x = 6 or x = 34.
b) x = 11 or x = 1.
c) x = 21 or x = 27.
3. a) x (1, 8)
b) x [2, 4]
4. |x 1 + y| < 2.78.

88

Unit 6

Polynomials
Content
6.1 Addition and Subtraction of Polynomials
6.2 Multiplication of Polynomials
6.3 Polynomial Division
6.4 Real Zeroes/Roots of Polynomials
6.4.1 Factorizing Polynomials

Introduction
Polynomials are algebraic expressions that deserve special attention because of their importance
in mathematics. In this Unit, we will learn how to add, subtract, multiply and divide polynomials. We will learn how to find zeroes/roots of a given polynomial, and extend the methods of
factoring quadratic expressions to polynomials of degree 3.

Objectives
By the end of this Unit, you should be able to:
identify polynomials and classify them.
add and subtract polynomials.
multiply polynomials.
divide polynomials.
89

use the remainder theorem to find the remainder when a polynomial is divided by a binomial.
factorize polynomials.
use the factor theorem appropriately.

Activities and Study Times


All the activities and studying this Unit should not take you more than seven (7) hours.

90

The term polynomial use to be taken to mean any algebraic expression which is a sum/difference
of algebraic terms. In recent decades, the term polynomial has got a new specific formal meaning,
and it does not refer to just any sum/difference of algebraic terms anymore. We now give its
definition.

Definition 6.0.1 An expression of the form


an xn + an1 xn1 + an2 xn2 + . . . + a1 x + a0 ,
where an , an1 , . . . , a0 are real numbers, with an 6= 0 and n a nonnegative integer, is called a
polynomial of degree n in x.

In Definition 6.0.1,
The nonnegative integer n (which is the largest power of x appearing in the polynomial)
is called the degree of the polynomial.
The numbers a0 , a1 , . . . , an are called the coefficients of the polynomial.
The number a0 is the constant coefficient or the constant term.
The number an , which is the coefficient of the highest power, is called the leading coefficient
and the term an xn is called the leading term.
For instance 8x5 + 5x4 2x3 + 3x2 17x + 3 is a polynomial of degree 5 in x, with the leading
coefficient 8, the leading term 8x5 , the constant coefficient 3, and the coefficients are 8, 5, -2, 3,
-17, 3. a2 + 52 a 1 is a polynomial of degree 2 in a.
Polynomials which consist of just a single term are called monomials. For example x2 and 17x7
are monomials.
Example 6.0.1 Determine whether the following algebraic expressions are polynomials. If they
are, state the degree and if they are not, explain why.
a) x5 + 2x3 + x
b)

x + x3

c) 2x3 + 1
d)

3 2
2 ix

+2

91

Solutions

a) The expression x5 + 2x3 + x is a polynomial of degree 5.


b)

x+x3 is not a polynomial because of the presence of the term


variables are suppose to be integers.

x. Powers of the polynomial

c) 2x3 + 1 is a polynomial of degree 3.


d)

3 2
2 ix

+ 2 is not a polynomial because the coefficient of x2 , 23 i is not a real number.




Remark 6.0.1 A nonzero constant is also a polynomial of degree zero, since a = ax0 , for a R
such that a 6= 0.
We now look briefly at how arithmetic operations are performed on polynomials.

6.1

Addition and Subtraction of Polynomials

Addition/subtraction polynomials is performed in the same way as addition/subtraction of any


other algebraic expressions (Section 2.1.1). That is, one adds/subtracts like terms.
For example,
(3x2 + 2x + 1) + (2x2 6x 3) = (3 + 2)x2 + (2 6)x + 1 3 = 5x2 4x 2.
Also,
(m3 1) (m3 2m2 + 1) = (1 1)m3 + (0 2)m2 + (1 1) = 2m2 2.
Note that, in general, any sum/difference of polynomials in x is also a polynomial in x.

6.2

Multiplication of polynomials

Multiplication of polynomials is done the same way as multiplication of any other algebraic
expressions (Section 2.1.2), i.e. multiply each term of the first polynomial with each term of the
second polynomial and simplify by collecting and adding/subtracting the like terms. In general,
the product of polynomial in x is also a polynomial in x.

92

Example 6.2.1 Simplify the following expressions.


a) (5x + 3)(4x + 7)
b) (2x3 x + 1)(x3 + 2x2 + 1)
Solutions
a) Remember, we multiply each term in the first polynomial with each term in the second
polynomial. So,
(5x + 3)(4x + 7) = 5x(4x + 7) + 3(4x + 7)
= 20x2 + 35x + 12x + 21
= 20x2 + 47x + 21

collecting and adding/subtracting like terms

b) We proceed in a similar manner as in a) to get,


(2x3 x + 1)(x3 + 2x2 + 1) = 2x3 (x3 + 2x2 + 1) x(x3 + 2x2 + 1) + 1(x3 + 2x2 + 1)
= 2x6 + 4x5 + 2x3 x4 2x3 x + x3 + 2x2 + 1
= 2x6 + 4x5 x4 + x3 + 2x2 x + 1

Note that in multiplication, we have used the laws of exponents (Theorem 2.1.1) a lot, to deal
with the powers. Therefore, we have the following remark.
Remark 6.2.1 In general, the degree of the product of two nonzero polynomials, by the laws of
exponents, is given by the sum of the degrees of the two polynomials.

6.3

Polynomial Division

Remember, in dividing real numbers, we use a Euclidean long division method. For instance if
you want to divide 87654 by 12, you proceed as follows.
 7304
12 87654
84000
3654
3600
54
48
6
93

The result of this division is that


87654
6
= 7304 +
12
12
or
87654 = 7304 12 + 6,
where 87654 is called the dividend, 12 the divisor, 7304 the quotient, and 6 the remainder.
Polynomial division is done using the same method, namely the Euclidean division method.
Lets consider dividing x2 6x 7 by x 1.


x1

x2 6x 7

First we divide the leading term of x2 6x 7 by the leading term of x 1 to get x.


x

x1

x2 6x 7

Then we multiply x 1 with x, and subtract the resulting polynomial from x2 6x 7.


x

x1

x2 6x 7
x2 + x
5x 7

Now we repeat the process, this time dividing the leading term of 5x 7 by the leading term
of x 1 to get 5.
x 5

x1

x2 6x 7
x2 + x
5x 7

Then we multiply x 1 with 5 and subtract the resulting polynomial from 5x 7.

94

x 5
x2


x1

x2

6x 7
+x
5x 7
5x 5
12

The process ends when, after subtracting, the degree of the resultant polynomial is less than the
degree of x 1. So, since the degree of x 1 is 1 and the degree of 12 is 0, the process ends
here, and therefore we write
x2 6x 7
12
=x5+
x1
x1
OR
x2 6x 7 = (x 1)(x 5) 12
In this equation, x2 7x 6 is called the dividend, x 1 the divisor, x 5 the quotient, and
12 the remainder.
In general, we therefore have the following Division Law of Polynomials.
Theorem 6.3.1 (Division Algorithm (Division Law of Polynomials)) If P (x) and D(x)
are nonconstant polynomials such that D(x) 6= 0, then there exist unique polynomials Q(x) and
R(x) such that P (x) = D(x)Q(x) + R(x), where either R(x) = 0 or the degree of R(x) is less
than the degree of D(x). i.e.
R(x)
P (x)
= Q(x) +
D(x)
D(x)
where P (x) is the dividend, D(x) the divisor, Q(x) the quotient, and R(x) the remainder.

This is just a formal way of putting together the procedure we performed above.

Example 6.3.1 Find the quotient and the remainder when


a) 2x3 + x2 3x 14 is divided by x 2.
b) x4 2x3 + 2x2 2x + 1 is divided by x3 + x2 x 1.

95

Solutions

a) We use the Euclidean long division method as follows:


2x2 + 5x + 7

x2

2x3 + x2 3x 14
2x3 + 4x2
5x2 3x
5x2 + 10x
7x 14
7x + 14
0

So,

0
2x3 + x2 3x 14
= 2x2 + 5x + 7 +
.
x2
x2

Therefore we have 2x2 + 5x + 7 is the quotient and the remainder is 0.


b) Proceeding as in a)
x3

x3 + x2 x 1

x4 2x3 + 2x2 2x + 1
x4 x3 + x2 + x
3x3 + 3x2 x + 1
3x3 + 3x2 3x 3
6x2 4x 2

So,

x4 2x3 + 2x2 2x + 1
6x2 4x 2
=
x

3
+
.
x3 + x2 x 1
x3 + x2 x 1

The quotient is x 3 and the remainder is 6x2 4x 2.



While theres really not much of a problem finding the remainder when one polynomial is divided by another, theres an even shorter method (than Euclidean long division) of finding the
remainder, when a polynomial of degree greater than or equal to 1 is divided by a polynomial
of degree 1. We state it in the following theorem.

96

Theorem 6.3.2 (The Remainder Theorem) If a polynomial P (x) is divided by (axb) then

the remainder is P ab .

Proof: The division law of polynomials states that P (x) = D(x)Q(x) + R(x). Since in this case
we have D(x) = ax b, we get P (x) = (ax b)Q(x) + R(x), where R(x) = 0 or the degree of
R(x) is less than the degree of ax b.
Since the degree of ax b is 1, R(x) must be some constant, say r. Therefore
P (x) = (ax b)Q(x) + r.

When x = ab , we have

  
b
b
= a b Q(x) + r = 0 Q(x) + r = r.
P
a
a
As required.

Theorem 6.3.2 means that to get the remainder when a polynomial in x is divided by a binomial
ax b, all that you need to do is substitute ab for x in the polynomial, and the resulting answer
will be the remainder.

Example 6.3.2
a) Using the remainder theorem, find the remainder when 3x2 + 6x 8 is divided by x + 3.
b) Using the remainder theorem, find the remainder when 8x3 + 4x + 3 is divided by 2x 1.
c) Given that the polynomial 2x3 + 3x2 + ax + b leaves a remainder of 7 when divided by x 2
and a remainder of 3 when divided by x 1, find the values of the constants a and b.

Solutions

a) Let P (x) = 3x2 + 6x 8. Then by the remainder theorem we know that the remainder is
given by P (3). So,
P (3) = 3(3)2 + 6(3) 8
= 27 18 8
= 1.

97

b) Let P (x) = 8x3 + 4x + 3. The remainder is given by P


P

1
2

. So,

 3
 
 
1
1
1
= 8
+3
+4
2
2
2
= 1+2+3
= 6

c) Let P (x) = 2x3 + 3x2 + ax + b. Now, since the remainder is 7 when P (x) is divided by x 2,
we have P (2) = 7. Further, since the remainder is 3 when P (x) is divided by x 1, we
have P (1) = 3. If we substitute these into the polynomial, we obtain the following pair
of equations.
2a + b = 17

(6.3.0.1)

a + b = 7

(6.3.0.2)

From equation 6.3.0.2 we have


a = 7 b

(6.3.0.3)

Substituting 6.3.0.3 into 6.3.0.1 we get


2(7 b) + b = 17 14 2b + b = 17
2b + b = 17 + 14
b = 3
b=3
Therefore a = 7 3 = 10.

For a) and b) above, you can verify that the remainders you obtained are correct by using the
longer route of Euclidean division.

6.4

Real Zeroes/Roots of Polynomials

Definition 6.4.1 Suppose P (x) is a polynomial. If c is a real number such that P (c) = 0 then
c is called the root or the zero of P (x), while x c is a factor of P (x).
For instance 2 and 5 are zeroes of the polynomial P (x) = x2 7x + 10 because P (2) = 22
7(2) + 10 = 4 14 + 10 = 0, and P (5) = 52 7(5) + 10 = 25 35 + 10 = 0. From Definition
6.4.1 we can also say that x 2 and x 5 are factors of Polynomial P (x) = x2 7x + 10.
98

The following Theorem helps us in locating zeroes for a given polynomial. Although it doesnt
really give a method of finding the zeroes, at least it points you where to look.

Theorem 6.4.1 (Intermediate Value Theorem of Polynomials) If P is a polynomial and


P (a) and P (b) have opposite signs then there exists at least one value c between a and b such
that P (c) = 0.

For instance, for the polynomial P (x) = 2x3 3x2 +2x4 we have P (1) = 23+24 = 3 < 0
and P (2) = 2(23 ) 3(22 ) + 2(2) 4 = 4 > 0. Therefore we know that this polynomial has a
zero between 1 and 2. It also follows that, between any two consecutive zeroes, the values of the
polynomial are either all positive or all negative.
To find these zeroes, all that you need to do is factorize the polynomial to obtain the factors,
and then obtain the zeroes from the factors. For those polynomials that cannot be factored,
there are numerical methods available for finding zeroes of polynomials, but we will not look at
them in this course.

6.4.1

Factorizing Polynomials

In this course, we will only consider factorization of polynomial of up to degree 3. Now, since
we already know how to factorize quadratic polynomials (polynomials of degree 2) from Section
2.1.4, we will use that as a basis for factorizing cubic polynomials (polynomials of degree 3).
If we suppose that P (x) is a polynomial of degree 3 and that it can be factorized, then we obtain
one of its zeroes by trial and error, which in turn will give us the first factor by applying the
following theorem.
Theorem 6.4.2 (The Factor Theorem) A polynomial P (x) has a factor ax b if and only

if P ab = 0.
Proof: We need to prove two things: I. () Given P (x) has a factor of ax b, we need to


show that P ab = 0. and, II. () Given P ab = 0, we need to show that ax b is a factor of
P (x).
I. () Let ax b be a factor of P (x). Then we have P (x) = (ax b)Q(x), for some Q(x). Then
  
  
 
b
b
b
b
P
= a b Q
=0Q
= 0.
a
a
a
a

99

II. () Let P

b
a

= 0. Applying the division law to P (x) with divisor ax b produces


P (x) = (ax b)Q(x) + R(x)



From the remainder theorem we get P ab = R(x). But since P ab = 0, then R(x) = 0.
Thus P (x) = (ax b)Q(x); which shows that ax b is a factor of P (x).

After obtaining the first linear factor, we divide the polynomial P (x) by the obtained linear
factor using the Euclidean long division method to get the quadratic factor. To complete the
factorization, the quadratic factor is then factorized using the methods discussed in Section
2.1.4.

Example 6.4.1 Factorize the following polynomials.


a) x3 x2 9x + 9
b) 2x3 x2 2x + 1

Solutions

a) Let P (x) = x3 x2 9x + 9. By trial and error, we see that 1 is a zero of this polynomial.
So, by the factor theorem, x 1 is a factor of this polynomial. Now, to get the quadratic
factor, we divide x3 x2 9x + 9 by x 1 as follows.
x2
x1

x3 x2 9x + 9
x3 + x2
9x + 9
9x 9
0

Which means that


x3 x2 9x + 9 = (x 1)(x2 9).
Factorizing x2 9 as in Section 2.1.4 we get
x3 x2 9x + 9 = (x 1)(x 3)(x + 3).

100

b) Let P (x) = 2x3 x2 2x + 1. By trial and error, we see that 21 is a zero of this polynomial.
So, by the factor theorem, 2x 1 is a factor of this polynomial. To get the quadratic
factor, we divide 2x3 x2 2x + 1 by 2x 1 as follows.
x2
2x 1

2x3 x2 2x + 1
2x3 + x2
2x + 1
2x 1
0

Which means that


2x3 x2 2x + 1 = (2x 1)(x2 1).
Factorizing x2 1 we get
2x3 x2 2x + 1 = (2x 1)(x 1)(x + 1).

The factor theorem can also be used to solve other problems involving polynomials, as the
following example demonstrates.

Example 6.4.2
a) Use the Factor theorem to determine whether x + 6 is a factor of P (x) = x3 + 4x2 27x 90.
b) The polynomial P (x) = x3 x2 + ax + b has a factor of x + 3, and leaves a remainder of 6
when divided by x 3. Find the values of the constants a and b.
Solutions
a) We substitute 6 for x in P (x) and see if it gives us 0.
P (6) = (6)3 + 4(6)2 27(6) 90
= 216 + 144 + 162 90
= 0
Since P (6) = 0, from the Factors theorem, x + 6 is the factor of P (x).

101

b) From the factor theorem we have P (3) = 0 and from the remainder theorem we get
P (3) = 6. Substituting these into the polynomial we get the following pair of equations,
which we solve as follows;
3a + b = 36
3a + b = 12
2b = 24
b = 12
Therefore a = 8.

Exercises 6.1
1. Find the quotient and the remainder when
a) x3 + 5x2 17x 21 is divided by x 4
b) x3 + 2x2 x + 2 is divided by x 1
2. Use the remainder theorem to find the remainder when 2x3 + 4x2 6x + 5 is divided by
x 1.
3. Given that the polynomial P (x) = x3 x2 + ax + b has a factor of x + 3, and leaves a
remainder of 6 when divided by x 3, find the values of the constants a and b.
4. Given that the polynomial P (x) = ax4 12x2 + bx 2 has a factor of x 1, and leaves a
remainder of 30 when divided by x + 2, find the values of the constants a and b.

6.5

Summary

1. Addition, subtraction and multiplication of polynomials is done in the same way as any
other algebraic expression.
2. The Euclidean long division process for polynomials ends when the degree of the remainder
is strictly less than that of the divisor.

102

6.6

Answers to Exercises in Unit 6

Exercise 6.1
1. If we denote the quotient with Q(x) and the remainder with R(x) then,
a) Q(x) = x2 + 9x + 19 and R(x) = 55.
b) Q(x) = x2 + 3x + 2 and R(x) = 4.
2. The remainder is 5.
3. a = 8 and b = 12.
4. a = 6 and b = 8.

103

Unit 7

Partial Fractions
Content
7.1 Case 1: Denominator with Distinct Linear Factors
7.2 Case 2: Denominator with Repeated Linear Factors
7.3 Case 3: Denominator with Irreducible Quadratic Factors
7.4 Case 4: Improper Rational Expressions

Introduction
Decomposing rational expressions into partial fractions comes in very handy in advanced mathematics, and it eases ones work especially in the Calculus courses. So, its very important to
know the methods involved. In this Unit, we will look at methods of decomposing rational
expressions of all types into partial fractions.

Objectives
By the end of this Unit, you should be able to:
identify rational expressions.
determine the form of partial fraction decomposition of any given rational expression.
decompose rational expressions into partial fractions.

104

Activities and Study Times


All the activities and studying this Unit should not take you more than six (6) hours.

105

In this Unit, we are going to use the phrase rational expressions a lot. So, it makes a lot of
sense to first define what rational expressions are.

Definition 7.0.1 Rational expressions are fractional algebraic expressions where both the numerator and denominator are polynomials.

For example
2
x+1

and

7 + 20x 3x2
2x2 11x 71

are rational expressions since in both expressions the top and the bottom are polynomials, while

x1
x2

and
x2 + 1
x+1
are not rational expressions. (Why?)
Now, in Section 2.1.1, we learned how to add/subtract more than one fractional expressions to
get a single fractional expression. In particular, we know how to add/subtract more than one
rational expression to get a single rational expression.
Lets consider the following equation:
1
x+3
2
+
=
.
x+1 x+2
(x + 1)(x + 2)
Weve added two simple rational expressions, and we have obtained a less simpler rational
expression.
Now, there are situations in mathematics (especially in calculus) where it would ease your
working to reverse this process. Therefore, what we want to do in this section is to start with a
(x)
rational expression PR(x)
(where P (x) and R(x) are polynomials) and ask ourselves a question:
What simpler rational expressions did we add and/or subtract to obtain this rational expression?.
These simpler rational expressions are called partial fractions and the process of obtaining
these partial fraction is referred to as partial fraction decomposition. This can be a fairly
long process, but it is quite simple if your knowledge on solving simultaneous equations is decent.
There are four cases to consider and we will discuss them in turns, starting with the simpler one.
P (x)
We consider a rational expression
, and we assume that in the first three of the following
R(x)
four cases the degree of P (x) is strictly less than the degree of R(x).

106

7.1

Case 1: Denominator with Distinct Linear Factors.

For the case where the denominator has distinct linear factors, we first factor the denominator
completely into its linear factors of the form ax + b. To decompose the given rational expression,
every linear factor in the denominator must have a corresponding partial fraction of the form
A
,
where A, a and b are constants.
ax + b
The task now is to determine the coefficients in the above terms, using algebraic methods, which
will be outlined in the following examples.
Example 7.1.1 Decompose the following expressions into partial fractions.
7x + 8
a)
.
(x + 4)(x 6)
b)
c)

x2

x+3
.
+ 3x + 2

9x2 + 34x + 14
.
(x + 2)(x2 x 12)

Solutions
a) The denominator is already factored into linear factors. The terms in the partial decomposition will then be as follows
7x + 8
A
B
=
+
(7.1.0.1)
(x + 4)(x 6)
x+4 x6
We then add the two rational expressions on the right side of equation 7.1.0.1 to obtain
the following,
7x + 8
A(x 6) + B(x + 4)
=
(x + 4)(x 6)
(x + 4)(x 6)
Equation 7.1.0.2 is only true if and only if the numerators are equal, i.e
7x + 8 = A(x 6) + B(x + 4) = (A + B)x 6A + 4B

(7.1.0.2)

(7.1.0.3)

Equating the coefficients of the powers of x in equation 7.1.0.3 we have: For the coefficient
of x1 we get 7 = A + B and for the constant coefficient (i.e x0 ) we get 8 = 6A + 4B.
We now have two linear simultaneous equations with two unknowns. Solving these for A
and B we get that A = 2 and B = 5. Therefore the partial fraction decomposition for this
rational expression is
7x + 8
2
5
=
+
.
(x + 4)(x 6)
x+4 x6
107

b) We first factor the denominator to get


x2 + 3x + 2 = (x + 1)(x + 2)
Since these are linear factors, the terms in the partial fraction decomposition will then be
the following
x+3
A
B
=
+
(7.1.0.4)
(x + 1)(x + 2)
x+1 x+2
We then add the two rational expressions on the right side of equation 7.1.0.4 to obtain
the following,
x+3
A(x + 2) + B(x + 1)
=
(x + 1)(x + 2)
(x + 1)(x + 2)

(7.1.0.5)

Equation 7.1.0.5 is only true if and only if the numerators are equal, i.e
x + 3 = A(x + 2) + B(x + 1) = (A + B)x + 2A + B

(7.1.0.6)

Equating the coefficients of the powers of x in equation 7.1.0.6 we have: For the coefficient
of x we get 1 = A + B and for the constant coefficient we get 3 = 2A + B. We now have
two linear simultaneous equations with two unknowns. Solving these for A and B we get
that A = 2 and B = 1. Therefore the partial fraction decomposition for this rational
expression is
2
1
2
1
x+3
=
+
=

.
(x + 1)(x + 2)
x+1 x+2
x+1 x+2

Remark 7.1.1 Note that this is exactly the equation that we had in our introduction.

c) We first factor the denominator to get


(x + 2)(x2 x 12) = (x + 2)(x + 3)(x 4)
Since these are linear factors, the terms in the partial fraction decomposition will then be
the following
9x2 + 34x + 14
A
B
C
=
+
+
(7.1.0.7)
(x + 2)(x + 3)(x 4)
x+2 x+3 x4
We then add the two rational expressions on the right side of equation 7.1.0.7 to obtain
the following,

9x2 + 34x + 14
A(x + 3)(x 4) + B(x + 2)(x 4) + C(x + 2)(x + 3)
=
(7.1.0.8)
(x + 2)(x + 3)(x 4)
(x + 2)(x + 3)(x 4)
108

Equation 7.1.0.8 is only true if and only if the numerators are equal, i.e
9x2 + 34x + 14 = A(x + 3)(x 4) + B(x + 2)(x 4) + C(x + 2)(x + 3)

(7.1.0.9)

If we let x = 4 in equation 7.1.0.9 we have that 294 = 42C. So, C = 7. Letting x = 2 in


equation 7.1.0.9 we have 18 = 6A. So, A = 3. Lastly, letting x = 3 we get 7 = 7B.
So, B = 1. Therefore the partial fraction decomposition for this rational expression is
9x2 + 34x + 14
3
1
7
=
+
+
.
(x + 2)(x2 x 12)
x+2 x+3 x4


7.2

Case 2: Denominator with Repeated Linear Factors.

In the case when the denominator has repeated linear factors (factors of the form (ax + b)n ,
where n N+ , n 6= 1), the term in partial fraction decomposition would be:
A1
A2
An
+
+ +
.
2
ax + b (ax + b)
(ax + b)n
Then we try to determine the coefficients. Consider the following example.
Example 7.2.1 Decompose the following into partial fractions.
2x + 3
a)
(x + 1)2
b)

7x2 29x + 24
(2x 1)(x 2)2

Solutions
a) We have a repeated linear factor for the denominator. Therefore we have the following term
in the decomposition
B
2x + 3
A
Ax + A + B
=
+
=
(x + 1)2
x + 1 (x + 1)2
(x + 1)2

(7.2.0.10)

Equation 7.2.0.10 is only true if the numerators are equal. i.e.


2x + 3 = Ax + A + B

(7.2.0.11)

Equating the coefficients in equation 7.2.0.11 we get A = 2 and 3 = A + B. So, B = 1.


The partial fraction decomposition for our rational expression is therefore
2x + 3
2
1
=
+
.
2
(x + 1)
x + 1 (x + 1)2
109

b) One of the linear factors of the denominator is repeated and the other is distinct. We
therefore have the following decomposition
7x2 29x + 24
A
B
C
=
+
+
2
(2x 1)(x 2)
2x 1 x 2 (x 2)2
Adding the rational expressions on the right hand side we get,
7x2 29x + 24
A(x 2)2 + B(2x 1)(x 2) + C(2x 1)
=
(2x 1)(x 2)2
(2x 1)(x 2)2
We now equate the numerator to obtain
7x2 29x+24 = A(x2)2 +B(2x1)(x2)+C(2x1) = (A+2B)x2 +(4A5B+2C)x+4A+2BC
Equating the coefficients for the powers of x we get the following system of linear equations
7 = A + 2B
29 = 4A 5B + 2C
24 = 4A + 2B C
From the first equation we have that A = 7 2B. We substitute this into the second and
third equations to get the following pair of linear equations:
1 = 3B + 2C
4 = 6B C
Solving these, we obtain that B = 1 and C = 2. So, it follows that A = 5. Therefore the
partial fraction decomposition for this rational expression is
5
1
2
7x2 29x + 24
=
+
+
2
(2x 1)(x 2)
2x 1 x 2 (x 2)2


7.3

Case 3: Denominator with Irreducible Quadratic Factors.

An irreducible quadratic expression is a quadratic expression that cannot be factored any further
(i.e. 4 < 0). If the denominator has a quadratic factor of the form (ax2 + bx + c)n , the term in
partial fraction decomposition would be:
A1 x + B1
A2 x + B2
Am x + Bm
+
+ +
.
ax2 + bx + c (ax2 + bx + c)2
(ax2 + bx + c)m
Then we determine the coefficients.
110

Example 7.3.1 Decompose the following rational expressions into partial fractions.
a)

8x2 12
.
x(x2 + 2x 6)

b)

3x3 + 7x 4
.
(x2 + 2)2

Solutions

a) First, you should note that x is a linear term as it can be written as x = x + 0, and that
x2 + 2x 6 is an irreducible quadratic term. Therefore the terms in the partial fraction
decomposition of this rational expression are,
A
Bx + C
8x2 12
= + 2
.
2
x(x + 2x 6)
x
x + 2x 6
Now we add the two rational expressions on the right hand side to get,
8x2 12
A(x2 + 2x 6) + (Bx + C)x
(A + B)x2 + (2A + C)x 6A
=
=
x(x2 + 2x 6)
x(x2 + 2x 6)
x(x2 + 2x 6)
Equating the numerators we find that
8x2 12 = (A + B)x2 + (2A + C)x 6A
For these to be equal, the corresponding coefficients must also be equal, and hence we have
the following system
8 = A+B
0 = 2A + C
12 = 6A
Solving which we obtain that A = 2, B = 6 and C = 4. Therefore the partial fraction
decomposition for this rational expression is
2
6x 4
8x2 12
= + 2
.
2
x(x + 2x 6)
x x + 2x 6
b) Again, the reader should note that we have a repeated irreducible quadratic factor, the terms
in the partial fraction decomposition should therefore be
3x3 + 7x 4
Ax + B
Cx + D
= 2
+ 2
2
2
(x + 2)
x +2
(x + 2)2

111

Similarly to the preceding examples we add the two rational expression on the right hand
side to get,
3x3 + 7x 4
(Ax + B)(x2 + 2) + Cx + D
Ax3 + Bx2 + (2A + C)x + 2B + D
=
=
(x2 + 2)2
(x2 + 2)2
(x2 + 2)2
We then equate the numerators and the corresponding coefficients to get the following
system of equations,
3 = A
0 = B
7 = 2A + C
4 = 2B + D
The solution to this system is, A = 3, B = 0, C = 1 and D = 4. Therefore the partial
fraction decomposition for this rational expression is
3x
x4
3x3 + 7x 4
= 2
+
.
(x2 + 2)2
x + 2 (x2 + 2)2


7.4

Case 4: Improper Rational Expressions.

P (x)
is called an improper rational expression if the degree
D(x)
of P (x) is greater than or equal to the degree of D(x). Now if you find that the degree of P (x)
is greater than or equal to the degree of D(x), then first use Theorem 6.3.1 to divide P (x) by
D(x) to get
P (x)
R(x)
= Q(x) +
,
D(x)
D(x)
A rational expression of the form

R(x)
into partial fraction as
D(x)
in either of the previous three cases (whatever the case may be). The procedure is illustrated in
the following example.

and then decompose the the resultant (proper) rational expression

Example 7.4.1 Decompose

x3 4x2 19x 35
into partial fractions.
x2 7x

112

Solutions
Since the degree of the numerator is greater than the degree of the denominator, we divide first
as follows,
x +3

2
3
2
x 7x
x 4x 19x 35
x3 + 7x2
3x2 19x
3x2 + 21x
2x
Therefore we have,
x3 4x2 19x 35
2x 35
=x+3+ 2
x2 7x
x 7x
2x 35
, whose terms in the partial fraction deWe then decompose the rational expression, 2
x 7x
composition are,
2x 35
2x 35
A
B
=
= +
x2 7x
x(x 7)
x
x7
Adding the rational expressions on the right hand side and equating the numerators we get,
2x 35 = (A + B)x 7A.
Equating the like terms we obtain,
2 = A+B
35 = 7A
The solution to this system is therefore A = 5 and B = 3. The partial fraction decomposition
for the original rational expression is therefore given by,
5
3
x3 4x2 19x 35
=x+3+ +
.
x2 7x
x x7

Remark 7.4.1 Its important to remember that partial fraction decomposition can only be done
once the degree of the numerator is strictly less than the degree of the denominator. Otherwise
divide first and then do partial fraction decomposition of the remainder term.

113

Exercises 7.1
1. Determine the form of the partial fraction decomposition of the following rational expressions
a)

7x2
.
(x + 2)(x2 + 5)2

b)

x3 + x + 12
.
(2x 1)3 x(x2 + 2x + 9)2

c)

x2 + x + 12
.
(3x 1)2 (2x2 + 3)2

2. Decompose the following rational expressions into partial fractions.


8x + 1
.
x4 1
x2
b)
.
(x 3)(x2 + 4)
a)

x2 + 1
.
x2 4
x3 + 3x2 2x + 1
.
d)
(x 1)(x + 2)
c)

7.5

Summary

When decomposing, first of all make sure that the degree of the numerator is strictly less
than the degree of the denominator. If it is not, use long division to divide first, and then
decompose the resulting remainder.
Make sure that the form of the wanted partial fractions is correct.
If you add/subtract the resulting partial fraction you should get the original rational
expression. If not, something is wrong with your partial fraction decomposition.

114

7.6

Answers to Exercises in Unit 7

Exercise 7.1
1. a)
A
Bx + C
Dx + E
+ 2
+ 2
x+2
x +5
(x + 5)2
b)
A
D
Ex + F
B
C
Gx + H
+
+ 2
+
+
+ 2
2
3
x 2x 1 (2x 1) (2x 1) x + 2x + 9 (x + 2x + 9)2
c)
B
Cx + D
Ex + F
A
+
+
+
3x 1 (3x 1)2 2x2 + 3 (2x2 + 3)2
2. a)
9
7
8x + 1
+

4(x 1) 4(x + 1) 2(x2 + 1)


b)
9
4x + 12
+
13(x 3) 13(x2 + 4)
c)
1+

5
5

4(x 2) 4(x + 2)

d)
x+2+

1
3

x1 x+2

115

Unit 8

Functions
Content
8.1 Definition of a Function
8.2 Arithmetic with Functions
8.3 Even and Odd Functions

Introduction
The concept function will keep coming up from this Unit onwards, probably for the rest of
your mathematical life. Now, if you dont understand this concept here, then the remainder
of you mathematical life will be meaningless. In this Unit we will therefore first of all give a
definition of a function and the definitions of all the related terms that will be used in relation
to a function. We will see how we can perform arithmetic with functions and how even and odd
functions behave.

Objectives
By the end of this Unit, you should be able to:
distinguish relations that are functions from the ones that are not functions.
find the images and the pre-images of real numbers under a given function.
find the domain and range of a function.
find the sum, difference, product and quotient of functions, and the domain thereof.

116

determine whether a given function is even, odd, or neither.

Activities and Study Times


All the activities and studying this Unit should not take you more than eight (8) hours.

117

8.1

Definition of a Function

In many disciplines in life, correspondences/relations exist between two sets of objects. These
exist in forms of tables, graphs or equations. Lets consider the following table:

Student Name

Test 1 Marks

Ananias T

71%

Darius M

35%

Festus G

50%

Iiyambo D

0%

Mwalengwa N

95%

Paulus S

67%

Shoopala F

50%

This table shows the marks given to 7 students after writing a Grade 11 Mathematics Test 1.
Note that you can have two different students in this class with the same mark (Festus G and
Shoopala F both are given 50%). But you cant give one student two different marks for the
same test. Furthermore, if you assume that a student who misses a test gets a 0% mark, then
you can also not have a student with no test mark, but you can have a mark thats not given
to any of the students (e.g. none of the students got a 20%). Correspondences/relations of this
type are called functions. We would call this process of giving marks to students a function from
the set of seven Grade 11 Mathematics students to a set of real numbers between 0 and 100.
Intuitively, this is how functions behave. We now give a formal definition of a function from one
set to another set.

Definition 8.1.1 Let X and Y be two sets. Then


(i) A function f from a set X to a set Y is a rule/mapping/relation that relates every element
x X to exactly one element y Y such that f (x) = y.
(ii) The set X is called the domain of f , denoted by Df , and the set Y is called the codomain
of f , denoted by Codf . Now, if a function f is explicitly defined by a formula/equation,
then Df can be taken as the set of all the real numbers (xvalues) for which
the function f is defined as a real number. i.e.
Df = {x R|f (x) = y for some real number y}.

118

(ii) The set of all the values of f (x) (outputs of f ) in Y , as x varies throughout Df , is called
the range of f , denoted by Rf . i.e.
Rf = {f (x) R|x Df }.
This implies that, the range of f is a subset of the codomain of f .
(iii) If f (x) = y for some x Df and y Rf , then y is called the image of x under f , while
x is called the pre-image of y under f .

For our example above, if we let our function f to be the marking process of test 1, then
Df = {Ananias T, Darius M, Festus G , Iiyambo D, Mwalengwa N, Paulus S , Shoopala F},
Codf = {y R|0 y 100},
and
Rf = {0, 35, 50, 67, 71, 95}.
The image of Iiyambo D under f is 0%, and the image of Shoopala F under f is 50%. On the
other hand, the pre-image of 71% under f is Ananias T, and the pre-images of 50% under f are
Festus G and Shoopala F.
If a Df then the image of a under f , f (a), is also referred to as the value of f at a, and it is
found by substituting with a for x in the function y = f (x), and then solving for y. If b Rf
then the pre-image of b under f is found by substituting with b for y in the function y = f (x)
and then solving for x.

Example 8.1.1
1. Let f (x) =

3
.
2x + 1

a) Find the image of 2 under f .


b) Find the pre-images of 1 and

1
5

under f .

2. Given that f (x) = 3, g(x) = 2x + 1, and h(x) = 3x2 4, evaluate and simplify each of the
following.

(i) g( 3)

(ii) f (2)

(iii) h(a + h)

119

(iv) f (a2 1).

Solutions

1. a) Remember, the image of 2 under f is the value of f (x) when x = 2. Therefore, we


substitute with 2 for x in f (x) to get,
f (2) =

3
3
= .
2(2) + 1
5

b) Pre-image for 1 under f ,


3
2x + 1
2x + 1 = 3
1 =

2x = 2
x = 1
Pre-image for

1
5

under f ,

3
1
=
5
2x + 1
2x + 1 = 15
2x = 14
x = 7

2. (i) g( 3) = 2( 3) + 1 = 2 3 + 1 4.46
(ii) f (2) = 3. f is the type of functions that are called constant functions, because
it has the same value for all real numbers.
(iii) h(a + h) = 3(a + h)2 4 = 3(a2 + 2ah + h2 ) 4 = 3a2 + 6ah + 3h2 4.
(iv) f (a2 1) = 3, for the same reason as in (ii).

Given any correspondence/relation, we must be able to determine whether it is a function or
not. For example, the relation that we used in the above example is a function because, every
real number (besides 12 ) is related to exactly one other real number. Consider the following
example.

120

Example 8.1.2 Determine whether each of the following equations defines y as a function of
x.
a) 9x y + 16 = 13
b) x = y 2
c) x = y 3

Solutions
We need to make y the subject of the formula in the equations, and then determine whether
each x values gives only one y value.
a) 9x y + 16 = 13 y = 9x + 3. Since each x value gives only one y value, the equation
defines y as a function of x.

b) x = y 2 y = x. Now, except for x = 0, every xvalue will give two yvalues, positive
and negative. Therefore, this equation does not define y as a function of x.
c) x = y 3 y =

x. This equation defines y as a function of x, for the same reason as in a).

We can also determine whether a given relation is a function or not by looking at its graph.
Graphing a function is very identical to graphing equations (i.e. plotting some points and then
joining them), so we will not give it so much attention. If you have difficulties graphing equations,
then you are advised to pick up any algebraic textbook and read it up.
The method of determining whether a given graph is a graph of a function or not is called the
vertical line test, and it is given in the following theorem.

Theorem 8.1.1 (The Vertical Line Test) A graph is a graph of a function if and only if no
vertical line intersects it at more than one point.

For example, consider the following graphs.

121

a) Graph of a function

b)

Not a graph of a function

The graph in a) is a graph of a function since our dashed vertical line (and any other vertical
line) intersects it at only one point, while the graph in b) is not a graph of a function because
our dashed line intersects it twice. Lets further look at the following example.
Example 8.1.3 Determine whether the described curve is a graph of a function.
a) A circle
b) An horizontal line
Solutions
a) A circle
y
6

'$
&%

This is not a graph of a function by the vertical line test, because the dashed vertical line
intersects our circle twice.
122

b) A horizontal line
y
6

a
-

This is a graph of a function by the vertical line test, because the dashed vertical line
intersects our horizontal line at only one point.

To indicate that f is a function from set X to set Y , we use the notation f : X Y . In this
notation X is the domain and Y is the codomain. Given any function, its very important to be
able to determine its domain and sometimes, its range as well. Now, to determine the domain
of a given function f (x), we need to find the set of all the real numbers for which the functions
f (x) is defined.
At this point, we only know of two cases when an algebraic function1 will not be defined: either
when you have a quotient of two algebraic expressions, in which case it will be undefined when
the numerator is zero; or when you have an even root of an algebraic expression, in which case
it will be undefined when the expression under the even root is negative. We will therefore
demonstrate how to find the domain of a given function in the following example.

Example 8.1.4 Determine the domains of the following functions.


3
2x + 1

b) f (x) = 2x 4
a) f (x) =

x+5
c) f (x) =
4
x2 9
1

An algebraic function is a function defined by an algebraic expression

123

Solutions
3
a) For the function f (x) =
, since we have a quotient of two expressions, it will be defined
2x + 1
for all the real numbers except those that will make 2x + 1 = 0. Therefore,

 

1
1
=R\
.
Df = {x R|2x + 1 6= 0} = x R|x 6=
2
2
Remember, the domain is defined as a set, therefore it should be given as a set.

b) For the function f (x) = 2x 4, we have a square root of an expression, which will only be
defined for those real number that will make 2x + 4 0. Therefore, we have,
Df

= {x R|2x + 4 0}
= {x R|2x 4}
= {x R|x 2}
= [2, ).

Note that this function is also defined when 2x + 4 = 0 because

0 = 0, a real number.

x+5
c) The function f (x) =
is a quotient of two expressions, and has an even root in the
4
x2 9

denominator. So, it will only be defined when 4 x2 9 6= 0 and x2 9 0. Combining


these two conditions, we get that our function will only be defined when x2 9 = (x
3)(x + 3) > 0. Solving this inequality we get,
Interval

(, 3)

(3, 3)

(3, )

Sign of (x 3)
Sign of (x + 3)

+
+

Sign of (x 3)(x + 3)

So, x2 9 > 0 for x < 3 or x > 3, therefore,


Df = {x R|x < 3 or x > 3} = (, 3) (3, ).

Note that 3 and 3 are not part of the domain because 4 x2 9 = 0 at those values.

Now that we know how to find the domain of a function, we turn our attention to the range.
To find the range, first write the function in the form y = f (x), then make x the subject of
the formula (having x on one side of the equation and everything else on the other), and then
determine the set of all the yvalues for which x will be defined as a real number and in the
domain of f . Consider the following example.

124

Example 8.1.5 Find the range of each of the following functions.


a) f (x) =

x2
10 x2

b) f (x) =

1
(x + 3)2

Solutions
x2
a) Rewriting this function in the form y = f (x) we get y =
. We now make x the
10 x2
subject of the formula as follows.
x2
10 x2
2
y(10 x ) = x2
y =

10y yx2 = x2
10y = x2 + yx2 = x2 (1 + y)
10y
= x2
1+y
r
10y
x =
1+y
Now, x will only be defined when

10y
0 and 1 + y 6= 0. Solving these we get,
1+y

Interval

(, 1)

(1, 0)

(0, )

Sign of (10y
Sign of x + 3
10y
Sign of
1+y

+
+

Therefore x is defined for y < 1 or y 0, and the range is


Rf = {y R|y < 1 or y 0} = (, 1) [0, ).
Again, since the range of a function is defined as a set, it should be given as a set.
b) We proceed as in a), by rewriting the function in the form y = f (x) to get y =

125

1
. We
(x + 3)2

then make x the subject of the formula to get,


y =
y(x + 3)2 =
(x + 3)2 =
x+3 =
x =

1
(x + 3)2
1
1
y
r
1

y
r
1
3
y

1
0 and y 6= 0. Solving these we get that x will be
y
defined when y > 0. So, the range is

Now, x will only be defined when

Rf = {y R|y > 0} = (0, ).



Exercises 8.1
1. Which of the following define y as a function of x?
a) x =

b) x =

y6

c) y =

2y + 1
y3

2. Which of the following curves are graphs of functions?


a) A P shaped curve.
b) The upper half of a circle.
c) An S shaped curve.
3. Let f (x) =

3
. For h 6= 0, find each of the following, if they exist.
2x + 1

a) the image of 1 under f .


c) f (a + h)

b) the pre-image of 3 under f .


f (a + h) f (a)
d)
h

4. Find the domain of the function f (x) =


5. Find the range of the function g(x) =

x2

1
.
x

1
.
(x + 1)2

126

8.2

Arithmetic with functions

Theres a way of combining functions using arithmetic (addition, subtraction, multiplication and
division). Before we see how functions can be combined, we will briefly state what is meant by
two functions being equal.
Definition 8.2.1 Given two functions f : A B and g : C D, we say that they are equal if
and only if A = C and B = D, and f (x) = g(x) for all x A(= C). This is written as f = g,
or sometimes as f g.

For example, consider the functions f (x) = x + 3 and g(x) =

x2 9
. Note that the function g
x3

(x 3)(x + 3)
x2 9
=
= x + 3, x 6= 3. Now, f (x) = g(x) for all
x3
x3
x R \ {3}, but Df 6= Dg (Df = R, but Dg = R \ {3}). Therefore f 6= g.

can be rewritten as g(x) =

Now we see how we can combine functions. If we have, say, two functions, we can form new
functions by combining them in different ways as follows.
Let f and g be functions. Then if x Df Dg , we have the following;

a) (f + g)(x) = f (x) + g(x).


b) (f g)(x) = f (x) g(x).
c) (f g)(x) = f (x) g(x).
 
f (x)
d) fg (x) =
, such that g(x) 6= 0.
g(x)

Example 8.2.1 Given that f (x) = x2 + 1 and g(x) =


a) (f + g)(5)
b) (f g)( 12 )
 
c) fg (4)

127

x2 1, find each of the following.

Solutions

a) Since this is a sum of two functions, we have the following,


(f + g)(x) = f (x) + g(x)
p
= (x2 + 1)( x2 1)
p
= x2 + 1 + x2 1
Therefore, (f + g)(5) = 52 + 1 +

52 1 = 25 + 1 +

24 = 26 +

24.

b) For multiplication we have


(f g)(x) = f (x) g(x)
p
= (x2 + 1) + ( x2 1)
q
1
1 2
So, (f g)( 2 ) = (( 2 ) + 1)( ( 21 )2 1).
Now, since

1
2

/ Dg , (f g)( 21 ) is not defined and hence

c) For division,
 
f
(x) =
g
=

So,

 
f
g

1
2

/ Df g .

f (x)
g(x)
x2 + 1

x2 1

17
42 + 1
= .
(4) =
2
15
4 1


8.3

Even and odd functions

Remember that, for real numbers, there are some that are even numbers, there are some that are
odd number, and there are others that are neither even nor odd. The same applies to functions.
The definitions of even and odd functions are however not the same as those for even and odd
number. The following are definitions for even and odd functions.
Definition 8.3.1
(i) A function f is even if f (x) = f (x) for every x in the domain of f .
(ii) A function f is odd if f (x) = f (x) for every x in the domain of f .
128

If a function f is such that f (x) 6= f (x) and f (x) 6= f (x) for some x Df , then f is said
to be neither even nor odd. Lets look at the following example.

Example 8.3.1 Determine whether the given functions are even, odd or neither.
a) f (x) = x3 x7
b) g(x) = 1 + x2
c) h(x) = x4 + 2x

Solutions
We substitute with x for x in each function and see whether we get the original function or
minus the original function.
a) f (x) = x3 x7
f (x) = (x)3 (x)7
= x3 (x7 )
= (x3 x7 )
= f (x)
Therefore the function is odd.
b) g(x) = 1 + x2
g(x) = 1 + (x)2
= 1 + x2
= g(x)
Therefore the function is even.
c) h(x) = x4 + 2x
h(x) = (x)4 + 2(x)
= x4 2x
6= h(x)

and

6= h(x)
Therefore h is neither even nor odd.
129


The graphs of even functions are symmetric with respect to the yaxis . That is, the left-hand
side of the yaxis is a mirror image of the right-hand side of the yaxis.
a)

b)
y

f (x) = x2

f (x) = |x|

@
@
@
@
@
@

The two graphs in figures a) and b) are graphs of even functions. (Verify this algebraically!!)
The graphs of odd functions are symmetric with respect to the origin . That is, a 180 rotation
of the graph about the origin gives the same graph.
d)

c)
y

f (x) = x3

f (x) = x3 x

The two graphs in figures c) and d) are graphs of odd functions. (Verify this algebraically!!)

130

Exercises 8.2
1. Let f (x) =

2 x and g(x) =

x. Find the following, if they exist.

a) The domain of f g
b) (f g)(3)
c) (f g)(0)
2. Determine whether the following functions are even, odd or neither.
a) f (x) = x4 4
b) g(x) = x2 2x + 1
1
c) h(x) = 3
x x
This is where we will leave it with functions, for now. More discussions on functions will come
next semester in Precalculus.

8.4

Summary

When finding the domain of a function, find the set of all the real numbers where the
function is defined.
When finding the range of a function, make x the subject of the formula, and put restrictions on y.
Always remember that both the domain and range are sets, each one specifying, respectively, the xvalues and yvalues for which a function is defined.
Finding the image of a certain value from the domain is just evaluating the function at
that particular value.

131

8.5

Answers to Exercises in Unit 8

Exercise 8.1
1. a) a function
b) not a function
c) a function
2. a) not a graph of a function
b) a graph of a function
c) not a graph of a function
3.
a) f (1) = 1
3
c)
2a + 2h + 1

b) x = 0
d)

6
(2a + 2h + 1)(2a + 1)

4. Df = R \ {0, 1}
5. Rg = {y R|y > 0}

Exercise 8.2
1. a) Df g = [0, 2]
b) (f g)(3) does not exist. (WHY?)

c) (f g)(0) = 2
2. a) Even
b) Neither
c) Odd

132

Unit 9

Trigonometry
Content
9.1 Angle Measure
9.2 Trigonometric Functions
9.3 Evaluating Trigonometric Functions
9.4 Graphs of Trigonometric Functions
9.5 Basic Trigonometric Identities
9.6 Trigonometric Equations

Introduction
Trigonometry was originally created by the Greek to help in the study of astronomy. Some time
earlier than 120 B.C., trigonometric ratios were already tabulated to enable the calculation of
a planets position. Nowadays, trigonometry has found even wider use in real life. It is used to
provide mathematical descriptions of vibrations, rotations, and periodic phenomena of various
kinds. In this Unit, we will look at two different ways of measuring angles. We will also give
an introduction to trigonometric functions and their graphs, look at some basic trigonometric
identities, and solve some basic trigonometric equations.

Objectives
By the end of this Unit, you should be able to:
convert angle measures from degrees to radians and vice versa.
133

recall the definitions of all six trigonometric functions.


evaluate trigonometric functions using calculators and the special triangles.
recall the shapes of the graphs of trigonometric functions.
recall the basic trigonometric identities and use them to prove further identities.
solve trigonometric equations

Activities and Study Times


All the activities and studying this Unit should not take you more than ten (10) hours.

134

Trigonometry is a subject based on the measurement of angles, hence the reason to start with
a discussion on angle measure.

9.1

Angle Measure

Previously, people were used to measuring angles in degrees. This is Babylonian measure of
angles, which has been used from as way back as before 115 B.C. In this course, well be using
a more natural method of measuring angles, called the radian measure1 . The radian measure
of an angle is defined as follows.
Definition 9.1.1 (Radian Measure of an Angle) Consider a circle of radius 1 with center
at the vertex of the angle. The radian measure of an angle is the length of the arc on the circle
that lies between the initial and the terminal sides of the angle.

terminal
side
of

radian
measure

@
R
'$
of

1 @
I initial
side
&%

of

Remember that the circumference C, of a circle with radius r is given by the formula
C = 2r.
It follows therefore that, the circumference of a unit circle (circle with radius 1)is 2. Since a
revolution angle is 360 , this means that the radian measure of the angle = 360 is 2. i.e.
2 radians = 360 .
Therefore


1 radian =

180

57.296 .

Hence, we have the following conversion rules from degrees to radians and vice-versa.
Conversion Rules for Radians

To convert degrees to radians, multiply with

180 .

To convert radians to degrees, multiply with

180
.

This method was invented by Thomas M


uir and James Thomson around 1870.

135

Lets consider the following two examples regarding conversions.

Example 9.1.1 Convert the following into radians.


a) 20
b) 30
c) 135
d) 270

Solutions

Remember that to convert from degrees to radians, we multiply with 180


. It is always preferred
to write your answer as a simplified fraction. So, we have the following,

a) 20 = 20

180

radians.

b) 30 = 30

180

radians.

c) 135 = 135

180

3
4

radians.

d) 270 = 270

180

3
2

radians.


Example 9.1.2 Convert the following into degrees.


a)

b)

5
6

radians
radians

c) 2
3 radians
d)

12

radians

Solutions
Converting from radians to degrees, we multiply with

a)

radians =

180

= 90 .
136

180
.

Therefore we have,

b)

5
6

radians =

5
6

180

2
c) 2
3 radians = 3

d)

12

radians =

12

180

= 150 .
180

= 120 .

= 15 .


Be it a result of laziness or something else, we dont know, but when writing angles in radians,
we normally leave out the units. No one will however get penalized for writing down the units.
If you choose to leave out the units, just keep it at the back of your mind that you are working
with radians.
PLEASE NOTE: From now on, unless otherwise stated, all our angles will be measured in radians.
In the above examples you have probably noticed that some angles are carrying a positive sign
while others are carrying a negative sign. The following is how we differentiate positive angles
from negative ones.
In a cartesian coordinate system, the standard position of an angle has a vertex at the origin
and initial side along the positive xaxis.

A counter clock-wise rotation produces a positive angle, whereas


A clock-wise rotation produces a negative angle.

a)

b)
6

initial side

+ve angle
?@
@ ve angle
@
@
@
@

initial side

137

The angle in figure a) is a positive angle because it is created by a counter clock-wise rotation
from the initial side, while the angle in figure b) is a negative angle because it is created by a
clock-wise rotation from the initial side. We therefore get the following picture.

2,

3
2

6
'$
- 0, 2

&%
3
2 ,

We do however prefer to work with positive angles. Therefore, if you encounter a negative angle,
try to give it in terms of its corresponding positive angle2 .

9.2

Trigonometric Functions

While at this point some of you might only be familiar with three trigonometric functions, we
actually have six trigonometric functions (the additional three being the reciprocals of the known
three).
1. Sine (sin)
2. Cosine (cos)
3. Tangent (tan) =

sin
cos

4. Cosecant (csc) =

1
sin

5. Secant (sec) =

1
cos

6. Cotangent (cot) =

1
cos
=
tan
sin

In terms of an angle , these functions are defined follows:


2

A positive angle whose terminal side is at the same position as your negative angle.

138

For Any Angle


y
6

P (x, y) s
@
@
r 
@
@

In the above diagram, if we let x be the xcoordinate and y be the ycoordinate of the point
P , and r be the distance of point P from the origin, then the trigonometric functions are defined
as;
y
r
, this implies that csc = , y 6= 0.
r
y
x
r
cos = , this implies that sec = , x 6= 0.
r
x
y
x
tan = , x 6= 0, this implies that cot = , y 6= 0.
x
y
sin =

The above definitions imply that, when = 0, , csc and cot are not defined, while when
= 2 , 3
2 , tan and cot are not defined.

For An Acute Angle


For acute angles, i.e. 0 < <

, we have the following;


2

















139

You must be familiar with the famous acronym SOH CAH TOA. We will use that here, in
reference to the above diagram to define trigonometric functions as follow;
b
c
sin = , this implies that csc = .
c
b
a
c
cos = , this implies that sec = .
c
a
tan =

b
a
, this implies that cot = .
a
b

So, in summary, for the three basic trigonometric functions, we have the following table.
Trig Function

In a right angled triangle ( acute, i.e. 0 2 )

For any angle

sin

opposite
hypotenuse
adjacent
hypotenuse
opposite
adjacent

ycoord
r
xcoord
r
ycoord
xcoord

cos
tan

In Terms of a Real Number


The value of a trigonometric function at a real number x is its value at an angle of x radians.
i.e. There is no difference between trigonometric functions of angles measured in radians and
those of real numbers. Now you see the importance of working with radians. You are therefore
advised to switch your calculator into the radian mode NOW. If you dont know how, read your
calculators accompanying manual, or get help from a friend.
We saw above that trigonometric functions are defined in terms of coordinates of a point on
a cartesian plane. Now, since coordinates of points have different signs in the four quadrants
of the cartesian plane, trigonometric functions also have different signs as you moves from one
quadrant to the next. The signs are as follows:
P (x, y)

y
6

P (x, y)

sin > 0
csc > 0

All are positive

Others < 0
-

P (x, y)

P (x, y)

tan > 0
cot > 0
Others < 0

cos > 0
sec > 0
Others < 0

140

Now that we have seen how trigonometric functions are defined and their signs in respective
quadrants, its time we combine these to determine how we can evaluate them at given angles.

9.3

Evaluating Trigonometric Functions

For any angle or real number, trigonometric functions can be evaluated using a calculator,
provided it is in radian mode. There are however some special angles that can be evaluated
using what is referred to as the Special Triangles. We will therefore first draw the special
triangles, and then use then to evaluate trigonometric functions.
Special Triangles




















From these triangles it follows that


1

sin =
6
2

3
sin =
3
2

3
cos =
6
2

cos

1
=
6
3

tan

tan

sin

1
=
4
2

1
=
3
2

cos

1
=
4
2


= 3
3

tan

=1
4

For the other three trigonometric functions, it is left to you to work them out for these special
angles.

Note that the angles above are all acute angles (i.e. angles between 0 and ). To evaluate
2
trigonometric functions for any other angles that are not acute, we make use of what is known
as the reference angles.
141

Reference Angles
Definition 9.3.1 Let be a non-acute angle. Then the reference angle R associated with
is the acute angle formed by the terminal side of and the xaxis.
Trigonometric functions for non-acute angles are evaluated using their reference angles as follows.

Let a, b > 0. Since the reference angle R is an acute angle (by definition), the point P (a, b) is
in the first quadrant. i.e.
y
6
sP (a, b)

r
R

Therefore we have,
b
sin R = ,
r

cos R =

a
b
and tan R = .
r
a

Now, if we have an angle whose terminal side is in the second quadrant, i.e.
< < , the
2
xcoordinate of our point will now be a and the ycoordinate remains b. So, = R
R = .
y
6

P (a, b)

s
@
@r
@
R @

142

So,
b
= sin R .
r
a
cos = cos( R ) = = cos R .
r
sin = sin( R ) =

tan = tan( R ) =

If < <

b
= tan R
a

3
, = + R R = .
2
y
6

R
r
P (a, b) s

So,
b
sin = sin( + R ) = = sin R .
r
a
cos = cos( + R ) = = cos R .
r
tan = tan( + R ) =

If

b
b
= = tan R
a
a

3
< < 2, then = 2 R R = 2 .
2

143

y
6

@ R
@
r@
@s

P (a, b)

So,
b
sin = sin(2 R ) = = sin R .
r
a
cos = cos(2 R ) = = cos R .
r
tan = tan(2 R ) =

b
= tan R
a

Example 9.3.1 Without using your calculator, evaluate each of the following.
a) cos

2
3

7
6

c) tan
4
b) sin

d) sec

11
6

Solutions
2
2

is in the second quadrant. So, R =


= .
3
3
3
2

1
cos
= cos = .
3
3
2

a) An angle of

7
is in the third quadrant. So, R =
= . Hence we have,
6
6
6
7

1
sin
= sin = .
6
6
2

c) The angle is an acute angle. Using one of the special triangles we get, tan = 1.
4
4
b) An angle of

144

11
d) For this problem we need to notice that
is an angle in the fourth quadrant. Therefore,
6
11

R = 2
= .
6
6
Therefore we have,
1
2
1
1
11
=
=
= = .
sec

11
3
6
3
cos
cos
2
6
6

From the special triangles and using the reference angle method, we get the following table of
values of trigonometric functions for the special angles.

sin

cos

tan

6
1
2
3
2
1
3

4
1
2
1
2

3
3
2
1
2

1
0
DNE

2
3
3
2
12

5
6
1
2

3
2
13

0
1
0

7
6
12

23
1
3

4
3

23
12

3
2

1
0
DNE

5
3

3
2

11
6
12

3
2
13

DNE stands for Does Not Exist or Undefined.


Remark 9.3.1 From our discussion above, we notice that trigonometric functions have positive
signs in two of the four quadrants and negative signs in the other two quadrants.

Exercises 9.1
1. Convert the following from degrees to radians.
a) 100
 
1
c)
2

b) 215
d)

2. Convert the following from radians to degrees.


a) 3 radians
b) 2 radians
3. Without using your calculator, evaluate each of the following.
5
4
7
b) cot
6
2
c) sin
3
a) cos

145

2
0
1
0

9.4

Graphs of Trigonometric Functions

Before we sketch the graphs of trigonometric functions, we need to briefly discuss a very important property that is exhibited by some functions, particularly, trigonometric functions.
Periodicity
A function f is said to be periodic if there is a positive number P such that f (x + P ) = f (x)
for every x Df . The least such number (if it exists) is called the period of f . This means
that a function f is periodic with period P if it repeats itself after every P units.
Graphs
We will use the table for values of trigonometric functions above to sketch the graphs of the
three standard trigonometric functions. The graphs for the reciprocal trigonometric functions
will be left to you to figure them out from the ones that will be given.
Graph of y = sin

y
6

3
2

3
2

From the graph of the function y = sin , we deduce the following properties of the sine function.
The function f () = sin is periodic, with period 2. i.e. sin( + 2) = sin .
The function f () = sin is an odd function because f () = sin = f ().
The range of the function f () = sin is the interval [1, 1].

146

Graph of y = cos

y
6

3
2

3
2

From the graph of the function y = cos , we deduce the following properties of the cosine
function.
The function f () = cos is periodic, with period 2. i.e. cos( + 2) = cos .
The function f () = cos is an even function because f () = cos = f ().
The range of the function f () = cos is the interval [1, 1].

Graph of y = tan
y
6

3
2

147

3
2

From the graph of the function y = tan , we deduce the following properties of the tangent
function.
The function f () = tan is periodic, with period . i.e. tan( + ) = tan .
The function f () = tan is an odd function because f () = tan = f ().
The range of the function f () = tan is R.
3
The function f () = tan is not defined when = , , . . ..
2
2

9.5

Basic Trigonometric Identities

The following trigonometric identities are very important. They are used mainly in simplifications of trigonometric expressions and in solving trigonometric equations.
Pythagorean Identities
The following identities are direct results of the Pythagorean formula for right angled triangles,
as it will become evident in the proof.
1. sin2 + cos2 = 1
2. tan2 + 1 = sec2
3. 1 + cot2 = csc2
Proof:

1. Consider the following diagram;


y
6
sP (x, y)

y
-

y
From the definitions of trigonometric functions we have that sin = y = r sin ,
r
x
and cos =
x = r cos . Now, from Pythagorean we know that y 2 + x2 = r2 .
r
148

Therefore r2 sin2 + r2 cos = r2 . Dividing through by r2 we get that


sin2 + cos2 = 1.

(9.5.0.1)

2. For this one, we divide through identity 9.5.0.1 by cos2 to get,


sin2
+1 =
cos2

1
cos2

tan2 + 1 = sec2 since tan =

sin
1
and sec =
cos
cos

3. We divide through identity 9.5.0.1 by sin2 to get,


1+

cos2
sin2

1
sin2

1 + cot2 = csc2 since cot =

cos
1
and csc =
sin
sin


Addition and Subtraction (Compound) Identities


These identities give formulae for expressing trigonometric functions of a sum or difference of two
angles. We will just state the identities in this course. You will meet the proofs in Precalculus
next semester.
cos( + ) = cos cos sin sin
cos( ) = cos cos + sin sin
sin( + ) = sin cos + cos sin
sin( ) = sin cos cos sin
tan( + ) =

tan + tan
1 tan tan

tan( ) =

tan tan
1 + tan tan

Double Angle Identities


The double angle identities are direct results of the addition identities we stated above. To see
how, just write 2 as ( + ), substitute into the addition identities, and then simplify. So, the
proofs for these identities is left to you as an exercise.

149

cos 2 = cos2 sin2


= 2 cos2 1
= 1 2 sin2

sin 2 = 2 sin cos


tan 2 =

2 tan
1 tan2

The above-stated identities can be used to prove other trigonometric identities. The general
method of proving identities is to choose either the left, or the right hand side (which ever is
convenient to start with), and show, by using known standard identities, that it can be rewritten
in the form of the other side. So, lets look at the next example.

Example 9.5.1 Prove the following identities.


a) tan + cot = sec csc
b) cos 2 = cos4 sin4

Solutions

a) We start with the Left-Hand Side (LHS), and show that it is equivalent to the Right-Hand
Side (RHS).
LHS = tan + cot
sin
cos
=
+
cos
sin
2
sin + cos2
=
cos sin
1
=
since
cos sin
1
1
=

cos sin
= sec csc

since

sin2 + cos2 = 1

1
= sec
cos

= RHS

150

and

1
= csc
sin

b) We will start with the RHS and show that it is equivalent to the LHS.
RHS = cos4 sin4
= (cos2 sin2 )(cos2 + sin2 )
= (cos2 sin2 ) 1
= cos 2

since

since

difference of squares.
cos2 + sin2 = 1

cos2 sin2 = cos 2

= LHS

Alternatively, you could prove identities by showing that either
LHS RHS = 0

or

LHS
=1
RHS

The following basic identities are also worth noting.


sin( + 2 ) = cos
sin( ) = sin
cos( + 2 ) = sin
cos( 2 ) = cos

9.6

Trigonometric equations

We will now use everything we learned in the previous sections to try and solve some trigonometric equations. Solving a trigonometric equation of the form sin = a, involves finding all the
values of , in the specified interval, that satisfy the equation. Quite often, students find this
exercise very difficult, therefore your attention is invited to this particular part.
Example 9.6.1 Without using your calculator, solve the following equations for , where 0
2.

3
a) cos =
2
b) tan( +

18 )

= 1

151

Solutions

a) We know from the sections above and in particular from Remark 9.3.1 that the cosine function
is positive in the quadrants I and IV. Therefore, our equation will have two solutions, one
from quadrant I and the second from quadrant IV.
Now, to get the solution in quadrant I, we use one of the special triangles, and we find the

angle whose adjacent side is 3 and hypotenuse 2, and we find that 1 = .


6
To get the solution in quadrant IV, we use 1 as our reference angle, R , and we find that

11
the solution in quadrant IV is 2 = 2 R = 2 =
. Therefore, the solutions to
6
6

11
our equation are 1 = , and 2 =
.
6
6
b) We follow the same procedure as in a), by noting that the tangent function is negative in
quadrants II and IV. First, we find our reference angle to be the solution to the equation

= =
. So,
tan = 1, which is . Therefore we have, in quadrant II, 1 +
4
18
4
4
3

25
1 =

=
.
4
18
36
In quadrant IV we have 2 +

7
7

61
= 2 =
. So, 2 =

=
.
18
4
4
4
18
36

Therefore, the solutions to our equation are 1 =

61
25
and 2 =
.
36
36

In the next example, we will need to use some trigonometric identities.

Example 9.6.2 Find the solutions to the equation 2 cos2 + sin = 2, where 0 2.

152

Solutions
We use the Pythagorean identity to rewrite the equation in terms of only one trigonometric
function, and we get,
2 cos2 + sin = 2
2(1 sin2 ) + sin = 2
2 2 sin2 + sin = 2
2 sin2 sin = 0
sin (2 sin 1) = 0

sin = 0 or 2 sin 1 = 0
1
sin = 0 or sin =
2

5
= 0, , 2 or = ,
6
6

Therefore, the solutions for our equation are; = 0,

,
6

5
,
6

and 2.

More trigonometry is coming your way in Precalculus, but for now, this is where we will leave
it.

153

Exercises 9.2
1. Without using your calculator, solve each of the following equations, for 0 x 2.
a) cos 12 x =

3
2

b) sin(x 2 ) = 1
c) csc2 x 4 = 0
2. Use Pythagorean theorem to solve for 0 .
a) 2 sec2 + 3 tan = 1
b) cos2 = 3 sin2
c) 7 sin2 5 sin + cos2 = 0
3. Prove that,
a) sin3 + sin cos2 = sin
cos2
= 1 + sin
1 sin
c) cos2 (1 + tan2 ) = 1
b)

9.7

Summary

Everything from now on should be in radian measure.


In the range 0 2, trig functions always have two solutions.

154

9.8

Answers to Exercises in Unit 9

Exercise 9.1
1.
5
radians
9

c)
radians
360

43
radians
362

d)
radians
180

a)

b)


540
2. a)

b) (180)


1
3. a)
2

b) 3

3
c)
2

Exercise 9.2

or x =
3
b) x =

c) x = or x =
6

1. a) x =

11
3
5
7
11
or x =
or x =
.
6
6
6

2. a) = 2.678 or =

3
4

5
or =
6
6

5
or = 0.340 or = 2.802
c) = or =
6
6
b) =

3. a)
LHS = sin3 + sin cos2
= sin3 + sin (1 sin2 )
= sin3 + sin sin3
= sin
= RHS

155

b)
LHS =
=
=
=
=

cos2
1 sin
cos2
1 + sin

1 sin 1 + sin
cos2 (1 + sin )
1 sin2
2
cos (1 + sin )
cos2
1 + sin

= RHS
c)
LHS = cos2 (1 + tan2 )
= cos2 sec2
1
= cos2
cos2
= 1
= RHS

156

Unit 10

Sequences
Content
10.1 The General Term
10.2 Recursive Definition
10.3 Arithmetic Sequence
10.4 Geometric Sequence
10.4.1 Infinite Geometric Sequence

Introduction
Infinite series can be used to find function values for functions that are difficult to compute.
The knowledge of sequences is very important in the definition of convergence or divergence
of a series. In this Unit, we give a brief discussion on sequences. We look at how sequences
are defined and how to find a specific term of a sequence for both arithmetic and geometric
sequences. We also take a brief look at infinite geometric sequence, and determining their sums.

Objectives
By the end of this Unit, you should be able to:
recall the definition of a sequence.
find the general term, given the first few terms of a sequence.
define sequences recursively.
157

find the common difference/ratio and the nth term of an arithmetic/geometric sequence.
find the nth partial sum of a given sequence.
find the sum of the infinite geometric sequence (if it exists).

Activities and Study Times


All the activities and studying this Unit should not take you more than eight (8) hours.

158

Lets consider the following Chemistry experiment. Chemical A is claimed to reduce the turbidity
of water faster than the other chemicals. So, a chemistry student is trying to see just how fast
exactly. She treats a certain amount of water with chemical A, and measures the turbidity of
water for successive days, starting 1 day after treatment. The readings will be in the following
form:
a1 , a2 , a3 , . . . , an , . . .
In the above readings, a1 represents the turbidity measurement taken 1 day after treatment,
while a3 represents the measurement taken 3 days after treatment, and so on.
Readings of this form are in what is referred to as a sequence. They represent an example
of a sequence. They are said to be ordered because they were taken in a specific order. The
term order does not necessarily refer to the values of an s. We then have the following formal
definition of a sequence.

Definition 10.0.1 A sequence is a set of real numbers, which is the range of a function whose
domain is the positive integers, or its subset. If the domain is the set of all the positive integers,
then the sequence is called an infinite sequence. If the domain is a subset of the positive
integers, then the sequence is called a finite sequence. The members of the sequence are called
the terms of the sequence, and the term corresponding to the positive integer n is the nth term
of the sequence, and it is denoted by a symbol like an . The nth term is also called the general
term of the sequence.

For example, consider a sequence given by the formula


an = n2 .
The expression an gives a formula of finding the nth term of the sequence. So, the first 5 terms
of this sequence are as follows:
a1 = 12 = 1
a2 = 22 = 4
a3 = 32 = 9
a4 = 42 = 16
a5 = 52 = 25

159

10.1

The General Term

The general term (the nth term) of a sequence can be found by looking at the first few terms,
and then determine how each term is obtained from the previous one so that one can identify
the pattern and make a guess.
Example 10.1.1 Find the general term for the sequences whose first five terms are given below.
a) 2, 4, 6, 8, 10, . . .
b) 12 , 13 , 41 , 15 , . . .
Solutions
a) These are the positive even numbers, and each term is obtained by multiplying positive
integers by 2, so the general term is an = 2n.
b) These are fractions, where the numerator is (1)n for the sign to be alternating, and the
(1)n
denominator is n + 1. The general term is therefore an =
.
n+1


10.2

Recursive Definition

Sequences can also be defined recursively. A recursively defined sequence is the one in which
some of the first terms are given, and then you are told how each succeeding term of the sequence
can be obtained from the preceding terms.
For instance,
a1 = 19,
an = an1 5.
is a recursively defined sequence, whose second term can be found from the first term as, a2 =
a1 5 = 19 5 = 14. The other succeeding terms can be found the same way.
Example 10.2.1
a) Consider the sequence defined recursively as follows:
a1 = 1,
an = nan1 .
Find a2 , a3 , a5 .
160

b) Give a recursive definition for each sequence. Then find the indicated term.
(i) 5, 8, 11, 14, . . .

Find the 6th term.

(ii) 1000, 100, 10, 1, . . . Find the 7th term.


Solutions
a) We find these terms successively by using a1 as follows;
a2 = 2a1 = 2 1 = 2;
a3 = 3a2 = 3 2 = 6;
a4 = 4a3 = 4 6 = 24.
b) (i) We look at the pattern and see that the numbers are increasing by 3 each time. So, the
sequence is defined by
a1 = 5,
an = an1 + 3.
To get the 6th term, we add 3 to 14 twice to get a6 = 20.
(ii) Again looking at the pattern we see that the terms are decreased by a factor of 10
each time. So, the sequence is defined by
a1 = 1000,
an = 0.1an1 .
To get the 7th term, we multiply 1 with 0.1 three times to get a7 = 0.01.


10.3

Arithmetic Sequence

In Example 10.2.1, the sequence in b)(i) has terms that are increasing by 3 each time. That
is, the difference between every two successive terms is constant (3). Sequences of this type,
where theres a constant difference between successive terms are referred to as the arithmetic
sequences. We give the definition as follows;
Definition 10.3.1 Let d be a real number. A sequence an is called an arithmetic sequence if
ai+1 ai = d for all i Z+
The number d is called the common difference for the sequence.
161

For instance, the sequence


4, 9, 14, 19, 24, . . .
is an arithmetic sequence with the first term a1 = 4 and the common difference d = 5.

Example 10.3.1 Find the common difference, d, for the following arithmetic sequences.
a) 1.5, 1.0, 0.5, 0, 0.5, . . .
b) 4, 3.99, 3.98, . . .

Solutions

a) We find the common difference by subtracting any two successive terms of the sequence. If
we subtract, say, the first one from the second one, we get
d = 1.0 1.5 = 0.5.
Subtracting any other two successive terms should give you the same answer.
b) Proceeding as in a), this time subtracting the second from the third gives,
d = 3.98 (3.99) = 0.01.

When we are provided with the first term, a1 , and the common difference, d, of the sequence,
we can find a2 by adding d to a1 . We can also find a3 by adding d to a2 once or by adding d
to a1 two times. In general, we have the following formula for finding the general term (the nth
term, an ) of our sequence.

Theorem 10.3.1 Given the first term, a1 , and the common difference, d, the nth term of an
arithmetic sequence is given by
an = a1 + (n 1)d, for any n 1.

162

Example 10.3.2
a) Find the fiftieth term of an arithmetic sequence whose first three terms are 2, 6, 10, . . ..
b) Given that the fifteenth term of an arithmetic sequence is 3 and the first term is 25, find
the tenth term.

Solutions

a) We get the common difference by subtracting the first term from the second term: d =
a2 a1 = 6 2 = 4. We then use the formula, with n = 50 to get
a50 = a1 + (50 1)(4) = 2 + 196 = 198.
b) We first solve the equation an = a1 +(n1)d for d, given that n = 15, a1 = 25 and a15 = 3.
3 = 25 + 14d,
d = 2.
Now, finding the tenth term we get:
a10 = 25 + (9)(2) = 7.

There might also be cases when well be interested in the sum of the first n terms of a given
arithmetic sequence, called the nth partial sum. We find it using the following formula.

Theorem 10.3.2 The nth partial sum, Sn , of an arithmetic sequence an is given by


Sn =

n
n
(a1 + an ) = [2a1 + (n 1)d],
2
2

since an = a1 + (n 1)d for any n 1.

For example, the sum of the first 100 positive odd integers is found as follows:
We note that a1 = 1, d = 2 and n = 100. Therefore a100 = 1 + 99 2 = 199. Thus,
S100 =

100
(1 + 199) = 10000.
2

Remark 10.3.3 The sum of the terms of a sequence is called a series.


163

Example 10.3.3 Find the 8th partial sum of the following arithmetic sequences.
a) 1.5, 1.0, 0.5, 0, 0.5, . . .
b) 4, 3.99, 3.98, . . .

Solutions

a) First we note that n = 8, a1 = 1.5, and from Example 10.3.1 a), d = 0.5. To find a8 we
use the formula an = a1 + (n 1)d and get, a8 = 1.5 + 7 (0.5) = 2. Therefore,
8
S8 = (1.5 + (2)) = 4 0.5 = 2.
2
b) We proceed like in a) by noting that n = 8, a1 = 4, and from Example 10.3.1 a), d = 0.01.
To find a8 we use the formula an = a1 + (n 1)d and get, a8 = 4 + 7 (0.01) = 3.93.
Therefore,
8
S8 = (4 + (3.93)) = 4 7.93 = 31.72.
2


10.4

Geometric Sequence

In Example 10.2.1 b)(ii), the terms of the sequence decrease by a common factor (10). Put
differently, the ratio of any two successive terms of the sequence is the same. Such sequences
are referred to as the geometric sequences. Thus, we have the following definition.
Definition 10.4.1 Let d be a real number. A sequence an is called a geometric sequence if
ai+1
= r for all positive integers i and r 6= 0
ai
The number r is called the common ratio for the sequence.

For instance, the sequence 3, 6, 12, 24, 48, 96, . . . is a geometric sequence, with the first term
a1 = 3 and the common ratio r = 2.

Example 10.4.1 Find the common ratio r for each of the following geometric sequence.
a) 2, 6, 18, 54, . . .
b) 1, 0.2, 0.04, 0.008, . . .
164

Solutions

a) To get r, we divide any two successive terms of the sequence. If we divide, say, the third
term by the second one, we get that
r=

18
= 3.
6

b) We divide the fourth term by the third one and obtain that
r=

0.008
= 0.2.
0.04


As was the case for the arithmetic sequences, its also possible to find the general term (the nth
term) of a geometric sequence when provided with the first term and the common ratio.

Theorem 10.4.1 Given the first term, a1 , and the common ratio, r, the nth term of a geometric
sequence is given by
an = a1 rn1 , for any n 1.

Example 10.4.2 Find the 10th term of the geometric sequence with the given first term and
common ratio.
a) a1 = 3 and r = 5
b) a1 = 64 and r = 21
Solutions
a)
an = a1 rn1 ,
a10 = 3 59
= 5859375.

165

b)
an = a1 rn1 ,
 9
1
a10 = 64
2


1
6
= 2 9
2
1
= 3
2
1
= .
8

For geometric sequences, the nth partial sum is given by the following formula.
Theorem 10.4.2 The nth partial sum, Sn , of a geometric sequence an with common ratio
r is given by
a1 (rn 1)
Sn =
, for any r 6= 1.
r1
For example, for a geometric sequence that satisfies the conditions that a1 = 5, and r = 2, its
sixth partial sum S6 is found as follows:
S6 =

5 63
5(26 1)
=
= 315
21
1

Example 10.4.3 Given that a geometric sequence has a3 = 32 and a6 = 4; find a1 , r, and the
8th partial sum of the sequence.

Solutions
Note that a3 = a1 r2 = 32 and a6 = a1 r5 = 4. Dividing the second equation by the first one we
get
a1 r5
4
=
a1 r2
32
1
1
Therefore we get r3 =
r = . Substituting for r in the equation for a3 we get that
8
2
 2
1
a1
= 32 a1 = 128.
2

166

Now, if we use the formula Sn =


S8 = 128

a1 (rn 1)
, we get
r1

(1 ( 12 )8 )
= 255.
1 12


10.4.1

Infinite Geometric Sequence

An infinite geometric sequence is a geometric sequence whose domain is the whole set of positive
integers.
An infinite geometric sequence does not have a sum when |r| 1 because rn increases without
bound (goes to infinity) as n increases. However, when |r| < 1, rn gets smaller and smaller as n
increases, and therefore the sum exists and is given by the following formula.
Theorem 10.4.3 If an is a geometric sequence with |r| < 1 and first term a1 , then the sum of
the infinite geometric series is given by
S =

a1
.
1r



X
2 n
For example, the infinite geometric series

is evaluated by first noting that a1 =


3
n=0
0
32 = 1 and r = 32 . Therefore


X
1
1
3
2 n
=
S =

2 = 5 = 5
3
1 ( 3 )
3
n=0

Example 10.4.4 Find S for the geometric sequence in Example 10.4.3, if it exists.

Solution
1
< 1, S exists. We know that a1 = 128, so
2
a1
128
128
S =
=
= 1 = 256.
1
1r
1 2
2

From example 10.4.3, since r =


Most of the problems that you may encounter should be solvable using the tools that we discussed
in this Unit. If you need further reading, any calculus book should be able to help you.

167

Exercises 10.1
1. Find the first four terms of the following recursively defined sequences.
a) a1 = 1, an = n2 an1
1
b) a1 = 1, a2 = 3, an = (an1 + an2 )
2
2. a) Find the sixth, twenty-fourth and the nth terms of the arithmetic sequence whose first
three terms are 8, 5, 2,
b) Given that the sixth and eighth terms of an arithmetic sequence are 14 and 20
respectively, find the twentieth term.
3. a) Find the tenth term of the geometric sequence whose first term and common ratio are
15 and 12 respectively.
b) Given that the first three terms of a geometric sequence are 8, 6, 92 , , find the ninth
and the nth terms.
c) Find the twentieth partial sum of the geometric sequence in b).


X
3 n
4. Find the sum of the infinite geometric series

.
5
n=0

10.5

Summary

1. The sum of an infinite geometric series is not defined for |r| 1.

168

10.6

Answers to Exercises in Unit 10

Exercise 10.1
1. a) 1, 4, 36, 576, . . .
b) 1, 3, 2, 52 , . . .
2. a) a6 = 7 and a24 = 61
b) a20 = 56
15
512
 n1
6561
3
b) a9 =
and an = 8
8192
4
c) S20 = 31.90

3. a) a10 =

4. S =

5
8

169

Unit 11

The Binomial Formula


Content
11.1 Factorial
11.2 The Binomial Coefficient
11.3 The Binomial Formula

Introduction
In this Unit, we will introduce the formula for expanding expressions of the form (a + b)n for n
a large positive integer, or n any rational number.

Objectives
By the end of this Unit, you should be able to:
find the factorial of any nonnegative integer.
evaluate Binomial coefficients for any rational number.
use the Binomial Formula to expand expressions of the form (a + b)n .
find a specific term in the expansion of the expression (a + b)n .

Activities and Study Times


All the activities and studying this Unit should not take you more than six (6) hours.
170

In Unit 2, we saw how we can expand algebraic expressions of the form (a + b)2 and (a + b)3 .
Generally, for (a + b)n , where n N, as n increases, expanding the expression becomes very
tedious. Therefore we need a formula to make expanding (a + b)n easier for larger n N. This
formula is given by what is called the Binomial Formula. Before we state the Binomial formula,
there are a few notations that we need to define because well need to use them later. First on
the list is the concept of the Factorial.

11.1

Factorial

Definition 11.1.1 (Factorial) If n N+ , then n!, which is read n factorial is given by


n! = n(n 1)(n 2) 2 1

That is, n! is the product of the first n positive natural numbers.


For example, 4! = 4 3 2 1 = 24, while 7! = 7 6 5 4 3 2 1 = 5040.
Remark 11.1.1 The factorial of 0 is defined as; 0! = 1.

Intuitively, n! can be looked at as the number of ways of arranging n different things in a row.
For instance, if you are asked to queue up three learners X, Y and Z, there are six possible ways
you can queue them:
XY Z

XZY

Y XZ

Y ZX

ZXY

ZY X

Note that the first student can be queued in 3 places, the second student can then be queued in
2 places, and the remaining student is then queued in the third place. Thus they can be queued
in (3 2 1) or 6 ways, which is 3!.
The second notation well take care of deals with what is known as the Binomial coefficient.

11.2

The Binomial Coefficient

Definition11.2.1
(The Binomial Coefficient) Let n, r N such that n r. The Binomial

n
coefficient
, which is read n choose r is defined as follows:
r
 
n
n!
=
.
r
r!(n r)!
171

 
n
Intuitively,
is the number of different ways you can choose r objects from a set of n objects.
r
 
n
Put differently, if X is a set such that |X| = n, then
= |{A X||A| = r}|.
r
 
7
Example 11.2.1 Evaluate
.
5
Solution
 
7
=
5
=
=
=
=

7!
5!2!
7654321
5 4 3 2 1 2 1
76543
54321
76
21
21

This means that there are 21 different ways of choosing 5 objects from a set of 7 objects.

Remark 11.2.1 You should convince yourself that for n, r N and r n,


  

n
n
=
.
r
nr
Note that the Binomial coefficient given in Definition 11.2.1 only makes sense for n N, since
n! is only sensible for n N. When n is negative, or a fraction, we have the following form of
the binomial coefficient.

Corollary 11.2.2 If n is any rational number then the binomial coefficient

 
n
is defined as
r

 
n
n(n 1)(n 2) . . . (n r + 1)
=
r
r!

You are advised to to convince yourself that for n N,

172

n!
n(n 1)(n 2) . . . (n r + 1)
=
.
r!(n r)!
r!

11.3

The Binomial Formula

We are now ready to state the Binomial Formula, which gives a way of expanding (a + b)n , and
also gives a way of finding the rth term of the expansion, where r n, without computing any
of the other terms. We state the theorem as follows.
Theorem 11.3.1 (The Binomial Formula) For any binomial a + b and any rational number
n,
 
 
 
 
n n 0
n n1
n n2 2
n 0 n
n
(a + b) =
a b +
a
b+
a
b + +
a b .
0
1
2
n
From Theorem 11.3.1 above, we can deduce that a particular term of an expansion is given as
follows.
Theorem 11.3.2 The (r + 1)st term of (a + b)n is given by
 
n nr r
a b
r

Example 11.3.1
1. Expand (x + y)4 .

10
2. Find the 4th term of 3x + y1
.
Solutions

1. Note that n = 4. Therefore we have that,


 
 
 
 
 
4 4
4 3
4 2 2
4
4 4
4
3
(x + y) =
x +
x y+
x y +
xy +
y
0
1
2
3
4
4! 4
4! 3
4! 2 2
4!
4! 4
=
x +
x y+
x y +
xy 3 +
y
0!4!
1!3!
2!2!
3!1!
4!0!
= x4 + 4x3 y + 6x2 y 2 + 4xy 3 + y 4
2. We note that n = 10, r = 3 and therefore r + 1 = 4. Thus we have that the 4th term of
the expansion is,



 3
10
10!
1
1
103 1
=
(3x)7 3 = 262440x7 3 .
(3x)
3
y
3!7!
y
y


173

Exercises 11.1
1. Evaluate the binomial coefficient

10
5

y)5 .
 1

1 10
3. Without expanding the binomial x 2 + x 2
, find the sixth term of its expansion.
2. Expand the binomial (2x

4. Find the term that contains y 8 in the expansion of (2x + y 2 )6 .

11.4

Summary

1. If one of the terms a or b in (a + b)n is negative, the terms of the expansion alternate signs.
2. The coefficients in the expansion of (a + b)n can also be obtained from the rows of Pascals
Triangle.

174

11.5

Answers to Exercises in Unit 11

Exercise 11.1
1.

10
5

= 252

2. (2x

p
p

y)5 = 32x5 80x4 y + 80x3 y 40x2 y 3 + 10xy 2 + y 5


5

3. 252x 2 x 2
4. 60x2 y 8

175

176

You might also like